SlideShare a Scribd company logo
1 of 43
Download to read offline
DISEASE: WHAT IS IT? HISTORY SYMPTOMS / PX CRITERIA (IF THERE IS) LABS DIAGNOSIS (INITIAL /ACCURATE) TREATMENT EXTRA NOTES!!!
SERONEGATIVE
SPONDYLOARTHROPATHIES
(Ankylosing spondylitis
Psoriatic arthritis
Reactive arthritis
Enteropathic arthropathy)
Ankylosing spondylitis:
- Inflammatory disorder of the axial skeleton and
the peripheral joints (Shoulder, hip)
- Distinct from most autoimmune disease in that it
is predominant in young men who are “under
the age of 40”
- MEN affected more than women by about 4:1
Reactive Arthritis:
- Inflammatory arthritis occurring after or during
a bacterial infection
- Two types which presents similarly:
- Post Diarrheal:
- occurs after infection with:
- Campylobacter
- Salmonella
- Shigella
- Reiter’s syndrome:
- occurs during or after: -
- nongonococcal urethritis
Ankylosing spondylitis:
N/A
Reactive Arthritis
History will be very important: for instance,
arthritis (>1 month) + urethritis has almost 99
% specificity for Reiter’s syndrome
Psoriatic Arthritis (PsA):
Ankylosing spondylitis:
- The primary complaint tends to
be persistent and chronic
refractory lower back pain
* worst in the morning and improves
throughout the day (like most
autoimmune arthropathies)*
PX:
- On examination patients will
have a loss of lumbar lordosis
which indicates the
characteristic fusion of lumbar
spine processes (this may make
the patient susceptible to spine
fracture)
Reactive Arthritis:
- Primarily affects:
- weight bearing joints
(knees, hips, lower back)
- Uveitis
Ankylosing spondylitis:
N/A
Reactive Arthritis:
N/A
Psoriatic Arthritis:
Classification criteria for psoriatic arthritis
(CASPAR):
If the patient has 3 or more points there is > 90
% sensitivity and almost 99 % specificity for
psoriatic arthritis
1. Family history of psoriasis:
a. Current (2 points)
b. Previous (1 point)
2. Psoriatic nail changes (1 point)
Ankylosing spondylitis:
N/A
Reactive Arthritis:
N/A
Psoriatic Arthritis (PsA):
N/A
Enteropathic Arthritis:
N/A
Ankylosing spondylitis:
The best first step is: based on clinical
symptom and on lumbar spinal XR
Schober’s test:
- A useful clinical tool for detecting a
loss of lumbar lordosis and therefore
possibly ankylosing spondylitis
- An increase of < 5 cm is considered a
positive Schober’s test and points to a
pathologic process
Reactive Arthritis:
- Can be made clinically.
- Should attempt to find the source of
the infection
- Patients with a history of diarrhea
within the last 2-6 weeks can be
considered to have:
Ankylosing spondylitis:
The best first step in therapy of patients with AS is:
- NSAIDs (Indomethacin; Ibuprofen)
- Physical therapy.
SECOND LINE AGENT:
- TNF inhibitors (infliximab, adalimumab, etanercept)
SURGERY:
- Hip replacement surgery
- Lumbar Spinal Osteotomy
Reactive Arthritis:
- NSAIDS are the cornerstone of therapy
- *Reactive arthritis and Reiter’s syndrome are self- limited*
Reiter’s Syndrome:
- a 3 months course of tetracycline may be of benefit
Derm symptoms:
- Topical corticosteroids (betamethasone)
Uveitis:
- Optho consultation
All seronegative spondyloarthropathies
share one thing in common: they all share
the presence of HLA B-27 antigen
(from the mutation on chromosome 6)
(Chlamydia trachomatis)
Psoriatic Arthritis (PsA):
- Inflammatory, erosive arthritis associated with
the skin condition psoriasis
Risk Factors:
- Personal and Family history of psoriasis.
- White > blacks, Hispanics
- MOA is around 35-50 years
Enteropathic Arthritis:
- Arthritis that is accompanied by and particularly
associated with flares of IBD
- About one in five people with Crohn's or
ulcerative colitis will develop enteropathic
arthritis
N/A
Enteropathic Arthritis:
N/A
- Dactylitis
- Low back pain
- Circinate balanitis
- Small, hard nodules on soles of
feet
- Possible continued symptom of
infection (diarrhea, dysuria)
PX:
1. Knee joint may be:
a. Warm
b. Red
c. fluctuant
2. In that case the best initial step
is to get a:
a. synovial analysis
3. Synovial fluid will be high for
WBCs,negative culture
Psoriatic Arthritis (PsA):
3. Dactylitis (1 point)
4. Radiographic changes (1 point) :
a. Osteophytes
b. joint erosion
5. Negative RF (1 point)
POOR PROGNOSIS:
1. Extensive skin involvement
2. A strong family history of psoriasis
3. Female gender
4. Disease onset at < 20 years of age
5. Expression of HLA-B27, -DR3 or -DR4
alleles
6. Polyarticular or erosive disease
Enteropathic Arthritis:
- a post diarrheal etiology
- Patient without a history of diarrhea
with a questionable sexual history
should consider a :
- throat and urogenital culture/
- PCR for Chlamydia
Psoriatic Arthritis:
Criteria based
Enteropathic Arthritis:
N/A
- Generally the treatment is ophthalmic corticosteroids (prednisolone)
Psoriatic Arthritis:
1. NSAIDs are the first line of therapy
2. Methotrexate is second line
3. Etanercept is approved for monotherapy
Enteropathic Arthritis:
1. NSAIDs
2. 5 – ASA derivatives (Sulfasalazine,pentasa)
3. Biologic medications, the TNF inhibitors
1. Plaque psoriasis (silver scaling,
erythematous plaques, guttate
lesions, pustules)
2. Psoriatic nail changes
3. Dactylitis (inflammation of the
digits, especially the DIPs)
Enteropathic Arthritis:
Axial or peripheral joint pain in a
patient with symptom of IBD
(Chronic diarrhea, cramping, mucus in
stool, tenesmus, oral aphthous ulcers)
Arthritis symptoms may precede the
IBD symptoms
N/A
OSTEOARTHRITIS Chronic, progressive, non- inflammatory ,
idiopathic, degenerative, joint disease
Primarily OA: Closely related to the aging process
Other arthropathies can predispose to OA
Cause of secondary OA:
- WEIGHT!!!!!
MOST COMMONLY AFFECTS:
- joints of the:
- Hands
- weight - bearing joints (knees, hips)
SYSTEMIC→
- Primary OA is NOT associated with any systemic
symptom
LOCATION→
- Hands (wrists, MCPs, PIPs, DIPs)
- Weight bearing joints (hips, knees)
INFLAMMATION→
- *JOINTS ARE NOT EXTERNALLY INFLAMED
(NO SIGNS OF INFLAMMATION)!!!*
Joint pain is most common presenting
complaint.
Patient tend to be older (50+)
Secondary OA Risk Factor is:
- Obesity
- Joint trauma
- Repetitive joint work
- and other arthropathies are risk factors
Primary Risk Factor:
- Usually Idiopathic
Patient’s will often complain of pain in
knees, hips, and hands
Absence of constitutional symptom
(no fever, out of the ordinary fatigue,
malaise,etc)
*REMEMBER: OA IS NOT AN
INFLAMMATORY CONDITION!!*
PX:
- Joint often appear normal,
externally.
- Decreased range of motion
(ROM)
- Crepitus→ osteophytes
- Nodes on DIPs and PIPs (both
nodes are present)
ALL LABS ARE NORMAL
*Abnormalities in labs are not
indicative of osteoarthritis*
The first step in diagnosis is X Ray of the
affected joints
Typical findings is a reduction in joint space
Therapy is palliative:
- Reduce pain
- Maximize functional capacity.
• NSAIDs are the mainstay of treatment !!! (ibuprofen, ketoprofen, meloxicam, diclofenac)
• Acetaminophen is the treatment of choice !!!
Occupational therapy
Weight reduction
Hot/cold compresses
Exercise
Physical therapy
CHRONICITY→
- chronic and progressive.
- Insidious onset.
- Worsens with activity, pain improves with rest
EVIDENCE OF TRAUMA→
- May occur secondary to traumas to specific
joints.
- Primary OA is not associated with trauma
RHEUMATOID ARTHRITIS Chronic inflammatory, systemic disease highlighted
by severe joint pain.
Women > Men
Progressive
Can dramatically reduce quality of life
Location :
- particularly in hands, but NOT in DIPS (distal
interphalangeal joint)
Systemic?
YESS!!!!! Most patients will have a history of:
- Fatigue
- Malaise
- Weakness prior to the onset of joint pain
- Fever
- Fatigue
- Malaise
Multiple systemic complications:
- Anemia of chronic disease (ACD)
- Felty’s syndrome (due to splenomegaly,
no pancytopenia)
- Baker’s cysts
Felty's syndrome:
- is a complication of long-standing
rheumatoid arthritis
- defined by the presence of three
conditions:
- Rheumatoid arthritis
- Splenomegaly
Technical diagnostic criteria
(ACR) Must fulfill at least 4 for diagnosis:
1. Morning stiffness of affected joints for > 1
hr X 6 weeks
2. Swelling of hand joints x 6 weeks
3. Symmetric joint swelling X 6 weeks
4. Subcutaneous nodules (Rheumatoid
nodules)
5. Positive rheumatoid factor (RF)
6. Deformities noted on radiography
Based on:
- Clinical
- Imaging
- Laboratory criteria
There is no one pathognomonic test
- positive RF
- Anti -cyclic citrullinated peptide
(anti-CCP) ( more sensitive but more
expensive)
*antibodies are highly suggestive*
The best initial diagnostic step is
radiography (XR is fine, CT is more
accurate)
*Xray first, CT more expensive*
Rheumatoid factor should be drawn as well
*Note: if patient has only one inflamed joint
or the inflamed joints are asymptomatic, a
synovial fluid test may be the best first step*
Metotrexate (MTX) *It can cause hepatic fibrosis*, with or without a biologic
agent
*NSAID, and steroids are not disease modifying*
Biologic agents:
- Primarily TNF- a inhibitors.
- Greatly reduce general inflammation
- Infliximab
- Etancercept
- Adalimumab
- Adverse effect:
- Immunosupression
- Before starting a patient on a biologics get a Tuberculosis test (PPD test) to
rule out latent TB
Newly diagnosed patients may be treated with an initial short- term course of:
- corticosteroids (prednisone), to bridge over, as MTX and biologics take effect
NSAIDs can be used for pain relief, but not alter disease course
Rheumatoid arthritis is associated with a high risk for morbidity and
premature death secondary to the earlier development of cardiovascular,
lung diseases and malignancy
Boutonniere deformity is a deformed position of the fingers or toes, in which
the joint nearest the knuckle (the proximal interphalangeal joint, or PIP) is
permanently bent toward the palm while the farthest joint (the distal
interphalangeal joint, or DIP) is bent back away (PIP flexion with DIP
hyperextension).
COMPLICATIONS:
- Adverse effects from medications
- Progressive reduction of quality of life
- Felty’s syndrome
- Baker’s cysts
- Carpal tunnel syndrome :
- Casting
- Immobilization
- Atlanto-axial subluxation :
- patient should be advised to caution anything that involves
intense cervical motion
- Symptom of impingement vary from hand/ foot paresthesias to
quadriplegia
- An abnormally low white blood cell
count.
Baker’s cyst:
- is a fluid-filled growth behind the knee.
- It causes a bulge and a feeling of
tightness.
- Also called a popliteal cyst.
- A Baker cyst sometimes causes pain.
- The pain can get worse with activity or
when fully
- Straightening or bending the knee
- Knee damage caused by a sports injury or
a blow to the knee can lead to a Baker's
cyst developing.
- Rheumatoid arthritis
- Osteoarthritis
- Gout
If a synovial fluid analysis is ran on a patient
With Rheumatoid Artheritis there will be:
- elevated WBCs not elevated enough
to suspect septic arthritis
- If suspected, first step is always to stabilize patient and get a
plain radiograph of the neck
*ACUTE PHASE REACTANTS: AFTER FELTY SYNDROME*
Although RA is more common in females, extraarticular manifestations of the
disease are more common in males !!!
The extra-articular manifestations of RA can occur at any age after onset
characterized by destructive polyarthritis and extra-articular organ
involvement, including the:
- Skin
- Eye
- Heart
- Lung
- Renal
- Nervous and gastrointestinal systems
Baker's cysts usually develop in people aged 30
to 70, although they can affect people of any
age, including children.
POLYMYALGIA RHEUMATICA ● ALWAYS SEEN IN >55
● TEMPORAL ARTERITIS CORRELATION
● IDIOPATHIC
● INFLAMMATORY DISORDER OF PAIN
● ASSOCIATED WITH PROXIMAL MUSCLES.
*PAIN*
● ABRUPT ONSET
● SELF LIMITED
*the key feature is pain, and not too much weakness*
Whites are affected more than blacks 2:1.
Woman affected more than men
There is a high correlation between polymyalgia
rheumatica and temporal (giant cell) arteritis.
This is a syndrome of the elderly (50+)
Nearly half of patients with temporal (giant cell)
arteritis have, or will have polymyalgia
rheumatica!!
Patinent will describe:
- General aches and pain
- Stiffness
- Especially of the upper arms
- Shoulders
- Hip girdle
The pain tends to be worst in the
morning
PX:
- Normal strength (however
exerting more strength may elicit
pain)
- Active range of motion is limited
by pain
Clinical and of exclusion
- The best initial test would be an ESR ,
which would be elevated * START THE
TREATMENT RIGHT AWAY PLEASE*
Oral corticosteroids, namely, prednisone
Patinet typically have a rapid response to corticosteroid (Then asses if patinet has teporal
arteritis via biopsy)
The steroids can be tapered down to the lowest effective dose
All patients should be assessed for symptoms related to temporal
arteritis, due to the high association
Patienst with symptomsof temporal arteritis will require:
- a temporal artery biopsy for confirmation and consultation with an
ophthalmologist.
However, the treatment (w/prednisone) is the same
Imagining does not play a significant role in diagnosing PR.
However, MRI has shown that the inflammation is more
around the bursa and tendon, rather, than inside the muscle
Itself
FIBROMYALGIA Chronic , non- inflammatory , “pain processing”
disorder
Syndrome of:
- Widespread pain
- Stiffness
- Fatigue
- Disrupted sleep
Often accompanied by:
- Mood or anxiety disorders
- Disturbances in memory and other somatic
disorders (Irritable bowel syndrome, overactive
bladder)
Increased incidence in patients suffering from
“autoimmune disorders”
Increased in patients who have suffered:
- Psychiatric or physical trauma (may start
disease)
Majority of the patients between the ages of 35 and 60
years
Symtom/ PX: Patient will appear tired
*Pain is illicitable from characteristic
“pressure points” which are mandatory
for diagnosis !!!*
A basic lab profile:
- CMP
- CBC
- TFT
- ESR
- A mental health exam
will be of use
ENTIRELY CLINICAL !!!
1. It is prudent to rule out or vigilant for
any other alternative/co- existing
disorders
2.
Symtomatic Treatment
All patients diagnosed with fibromyalgia should be prescribed a medication for the pain
The most common drug used is a TCA *for pain killers* (amitriptyline)
Recently, gabapentin and pregabalin have been used *More effective but more expensive*
Sleep medications:
- Zolpidem (reduces fibromyalgia fatigue, but not fibromyalgia pain)
*Patient requires psychiatric evaluation/ consultation)
Patients often complaints of “ hurting all over” “feeling like I always have the
flue”, persistent fatigue, co- existence of other somatic problems such as IBS
or overactive bladder
TRIGGER POINTS:
- 18 points (9 pairs) tend to be painful when pressed, and may spread
pain to other body parts.
People with fibromyalgia have pain in at least 11 of these tender points when
a doctor applies a certain amount of pressure
Women are more likely to be diagnosed with
Fibromyalgia
May be difficult to diagnose and is often a diagnosis of
exclusion !!!
SYSTEMIC LUPUS
ERYTHEMATOSUS (SLE)
- Chronic, inflammatory, multisystemic disease in
which arthritis is one of a myriad of possible
symptoms and presentations
- Idiopathic
- UNKNOWN ETIOLOGY AND SOME GENETIC
PREDISPOSITION
- Blacks and Hispanics are affected at higher rates
than whites
- Women are affected more than man by a 9:1
ration
- Age of onset:
- < 16 years (20%)
- >16 to 55 years (65%)
- > 55 years (15%)
Major causes of mortality:
- Infections and nephritis (early)
- Atherosclerosis (late)
Genetics:
- HLA-A1
- HLA-B8
Younger female, often of color presenting with
facial rash and/or joint pain
Be sure to get OB history
Anti - RO antibodies
- Fever
- Chills
- Headache
- Fatigue
- Malaise
- Weight loss
Renal:
- Hematuria
- Protenuria
Skin:
- Malar “Butterfly” rash
- Photosensitivity rash
Cardiac:
- Myocarditis
- Pericarditis
- Endocarditis
- Atherosclerosis
- Pancarditis
CNS:
- Conginitive dysfunction
DIAGNOSTIC CRITERIA FOR SLE
(SOAP BRAIN MD)
Four diagnostic criteria satisfies
diagnosis
S- Serositis
O- Oral Ulcers
A- Arthritis
P- Photosensitivity *avoid sunlight =,
because it can aggravate and cause
B- Blood D/O
R- Renal Abnormalities
A- ANA+ ⇒ not Gold Standard
I- Immunological phenomenon (anti-
Sm, AntidsDNA) *GOLD STANDARD*
Ultimately a diagnosis based on criteria and
supported by laboratory evidence
the best initial diagnostic test is ANA
*ANA IS NOT THE MOST ACCURATE*
Patients who are ANA + should get
Anti-dsDNA and Anti- Sm serologies, as
they are more specific (HALLMARK)
Focused on controlling symotoms
Arthralgia/pleuritis/ inflammation:
- NSAIDs (Ibuprofen, ketorolac, etc)
Rash:
- Corticosteroid topical ointment (betamethasone)
*Preventative care and corticosteroids (prednisone) should be used when
symptoms become frequent and certainly if patient develops renal
insufficiency*
In severe cases (severe lupus nephritis, heart/lung/ CNS involvement)
patients should be started:
- Cytotoxins (azathioprine, cyclophosphamide)
- HLA-DR
*Classic triad:
- Fever
- Arthralgia
- rash (malar, discoid, photosensitive)
Pregnancy and SLE:
- At high risk for spontaneous abortion
- This is a manifestation of anti- phospholipid
antibody syndrome
- All pregnant woman should be screened during
pregnancy for Anti- Ro antibody
- Babies born to Ro+ mothers are at risk for
neonatal lupus , which typically manifests with a
complete heart block
- All drug should be stopped during first
trimester, After first trimester patient should be
started on lowmolecular weight heparin and
aspirin
NEONATAL LUPUS
Pulmonary:
- Pleurisy
Musculoskeletal:
- Myalgias
- Arthralgias
- Arthritis
Hematologic:
- Anemia
- Leucopenia
- Thrompicytopenia
- Pancytopenia
Lymphatic system:
- Lymphadenopathy
OB/GYN:
- History (Spontaneous Abortions)
N- Neurologic symptoms not explained
otherwise
M- Malar Rash *Butterfly*
D- Discoid Rash
In emergent flares (RP GMN, severe neuro symptom):
- iv corticosteroids and cytotoxics should be used
Belimumab is the one biologic that is approved for use in SLE
Clinical Manifestation:
- Cardiac:
- Conduction defects
- Complete heart block rhythm
abnormalities
- Dermatologic:
- Erythematous papules
- Annular plaques
- Hematologic:
- Thrombocytopenia
- Leukopenia
- Anemia
- Hepatic:
- Hepatitis
TREATMENT:
- Involves:
- Surgical implantation of a pacing device
- Symptomatic control
- Generally with hydrochloroquine and IV
or topical steroids
NSAIDS SHOULD BE AVOIDED => MAY DEVELOP
RAYE’s SYNDROME
RESOLVES BY 6-8 MONTHS OF AGE!!
MYOSITIS
( POLYMYOSITIS &
DERMATOMYOSITIS)
Idiopathic (slow- onset, progressive), autoimmune,
inflammatory disease of skeletal (and
occasionally cardiac) muscle
Idiopathic myositis (Polymyositis, Dermatomyositis,
Inclusion body myositis):
- Age of onset is around 45-60 years
- Increased incidence with other autoimmune
diseases
- Best first diagnostic step is:
- Creatine phosphokinase (CPK) and
Aldolase
- Most accurate test is:
- biopsy and it is absolutely necessary to
establish the diagnosis
POLYMIOSITIS & DERMATOSIOTIS:
Women are affected more than men 2:1,
Blacks more than whites 5-8:1
*For inclusion body myositis men affected more
than woman*
POLYMYOSITIS:
1. Idiopathic, autoimmune and inflammatory
2. Affects proximal muscles most dramatically
3. Symmetrical
4. Distal muscles are relatively conserved
5. Ocular muscles are NEVER involved
6. Prognosis is good: main complications are:
a. dysphagia
b. interstitial lung disease
*A high creatinine kinase result would suggest
Polymyositis, CK is released when muscles breakdown
which occurs in polymyositis*
*Symmetrical muscle weakness affecting big muscles
is seen in polymyositis*
Polymyositis:
- Progressive Weakness
- Problems with ADL (activities of
daily living) including:
- brushing hair
- getting out of a chair
- holding head up
- carrying objects
- lifting arms up over head
Symptoms will be worst in the
morning, and improve throughout the
day
May present:
Weight loss
Fever
Hyperhydrosis
Generalized hyperhidrosis
Chronic pain
POLYMYOSITIS:
1. best first step in the
management/diagnosis:
a. CPK and aldolase levels
*These are markers for
muscular inflammation*
2. The most accurate test is:
a. muscle biopsy *Do this if
CPK/Aldolase are high*
3. It is judicious to order titers for Anti-
Jo1
a. Patient + for this antibody are
at increased risk to develop
interstitial lung disease
POLYMYOSITIS:
Best initial treatment for polymyositis is:
- Corticosteroids, (prednisone), If prednisone alone is not effective, cytotoxics or
biologics may be added
ADULT POLYMYOSITIS: Rituximab
DERMATOMYOSITIS:
Rituximab in the Treatment of Refractory Adult and Juvenile Dermatomyositis
DERMATOMYOSITIS:
Idiopathic, autoimmune and inflammatory disease
Also affects the skin.
1. Affects proximal muscles most dramatically
2. Symmetrical
3. Distal muscles are relatively conserved.
4. Ocular muscles are NEVER involved
INCLUSION BODY MYOSITIS:
Hereditary or acquired inflammatory and pathologic
disorder of muscle.
Inclusion body myositis may be asymmetrical
Affect distal muscles (hands, fingers) more notably
Fatigue
Dyspnea
Malaise
DERMATOMYOSITIS:
*same as polymyositsi*
- Progressive Weakness
- Problems with ADL (activities of
daily living) including:
- brushing hair
- getting out of a chair
- holding head up
- carrying objects
- lifting arms up over head
May present:
Weight loss
Fever
Hyperhydrosis
Generalized hyperhidrosis
Chronic pain
Fatigue
Dyspnea
Malaise
1. The best first step is:
a. CPK and aldolase levels.
2. The most accurate test is:
a. muscle biopsy (do this if
CPK/Aldolase are high)
Inclusion Body Myositis:
- Is refractory to steroids and immunosuppressants.
- Not deadly
-
- Early stages of disease can be confused w/ amyotrophic lateral sclerosis.
The muscle biopsy will confirm Inclusion body myositis
Unlike Inclusion body myositis amyotrophic lateral sclerosis is fatal
Affects men more than woman In addition, dermatomyositis will
present with the characteristic
heliotropic rush (face, eyelids,
sunexposed areas)
2 SIGNS & 1 RASH :
GOTTRON’s SIGN:
- purple/pink papule over the
Metacarpophalangeal (MCP) and
proximal interphalangeal (PIP)
joints.
HELIOTROPE RASH:
- purple/pink rash around the
eyes - eyelids may be swollen.
SHAWL SIGN:
- rash over the upper back,
shoulders and a V on the chest
INCLUSION BODY MYOSITIS:
INCLUSION BOSY MYOSITIS
1. The best initial diagnostic step:
- Insidious asymmetrical
weakness:
- Dysphagia
- Distal and proximal
weakness,
- Droopy head
- Facial muscle weakness
- No ocular involvement
Px: Decreased tendon reflexes are
common
a. CPK and aldolase
2. The most accurate test is:
a. Biopsy
*Electromyography (EMG) may be ordered*
WEGENER’S GRANULOMATOSIS Auto-immune inflammatory systemic disease of the
SMALL VESSELS!!
Commonly implicated by C- ANCA
Involves: Organ System, but most prolly:
Respiratory tract
Skin
Kidneys
History of chronic sinusitis that is refractory to
therapy
Refractory chronic sinusitis
“Saddle nose” deformity
Hemoptysis
Dyspnea
Polyarticular arthritis
Palpable purpuric rash
N/A Elevated ESR
Elevated CRP
Normocytic Anemia
Abnormal UA
The best initial test: an ANCA (c-ANCA) titer
The most accurate test is: Renal Biopsy
*shows granulomas*
Prednisone and cyclophosphamide are the mainstays of therapy
After 18 months, Cyclophosphamide
( which cannot be used more than 18 months) can be replaced with Methotrexate or
Azathioprine
Among the vasculitis in that the upper respiratory symptoms tend to take
central stage
Note: the lungs will always be involved even if the patient is asymptomatic.
CXR or CT (more accurate but may not be necessary) will show nodules
*ONLY CYCLOPHOSPHAMIDE REPLACES*
BONUS: What is an adverse effect of Cyclophosphamide?
Patient’s who are on cyclophosphamide therapy should get prophylaxis for
Pneumocystis carinii pneumonia
• TMP/SMX or (dapsone or atovaquone)
MICROSCOPIC
POLYANGIITIS
Auto- immune inflammatory systemic disease of the
SMALL VESSELS!!
Commonly implicated by P-ANCA
Can involve any organ system, but most
prolly involved are:
Skin
Lungs
Kidneys
Neuromuscular system
*MULTISYSTEMIC*
Most patient’s will describe a history of
frequent:
Muscle aches and pains
Fatigue
Weakness
Malaise
Weight loss
Hemoptysis
Mononeuritic multiplex:
(Inflammation of two or more
nerves, typically in unrelated parts of
the body)
Palpable purpuric rash:
N/A
Elevated ESR
Elevated CRP
Normocytic Anemia
Abnormal UA
The best initial test: an ANCA (p- ANCA)
titer
The most accurate test is biopsy
NOT IN PPT????? Note: Microscopic polyangiitis can be conclusively
differentiated from Wegener’s granulomatosis on
biopsy
No granulomas SEEN
POLYARTERITIS NODOSA Autoimmune inflammatory systemic disease of the
MEDIUM VESSELS, particularly those in the muscle
Can involve any organ system, but uniquely PAN spares
the lungs (NEVER INVOLVED)
Most commonly involved:
Skin
Kidney
Nerves
N/A Neuro/neuromuscular symptoms are
most prominently present in 70% of
patient which includes:
Peripheral neuropathy
(pins and needles)
Transient loss of vision
Patient must have 3 of the following 10 criteria :
1. Weight Loss of 4 KG or more
2. Livedo reticularis (Mottled skin)
3. Testicular pain or tenderness
4. Myalgia or leg weakness/tenderness
Elevated ESR
Elevated CRP
Normocytic Anemia
Abnormal UA
Best initial diagnosis & most accurate test is
a biopsy
*This can be done at a skin ulcer, at the
sural nerve or skeletal muscle.*
*If a biopsy cannot be undertaken,
angiogram is ok (i.e renal angiogram).
Prednisone and Cyclophosphamide *Postprandial GI pain is not uncommon*
*Hypertension secondary to renal factors is common too (restricted arterial
flow to kidneys activates RAAC*
*In patients with HBV & HCV, cyclophosphamide CAN NOT BE USED!!!*
Cyclophosphamide may be substituted with Methotrexate or Azathioprine in
18 months (remission
GI tract (Postprandial abdominal pain)
Associated with:
Hep B
Hep C
Mononeuritis multiplex
(Inflammation of two or more
nerves, typically in unrelated parts of
the body)
The skin rash is similar to WG and MPA,
however it may also include livedo
reticularis and Raynaud phenomenon
(due to the fact that this is a medium
vessel disease)
Significant weight loss
Men: testicular pain or tenderness
5. Mononeuropathy or polyneuropathy
6. Diastolic BP> 90 mmHg
7. Presence of HBsAg in serum
8. Arteriogram demonstrating aneurysms
or occlusions of visceral arteries
(transient loss of vision)
9. Biopsy of small or medium- sized artery
showing polymorphonuclear leukocytes
(PMNs) in wall
FOR CONCLUSIVE DIAGNOSIS, THE PATIENT
MUST HAVE BIOPSY!!
Angiogram will show multiple aneurysms*
CHANG-STRAUS SYNDROME Autoimmune, inflammatory systemic disease that is
uniquely associated with asthma and eosinophilia
Like Wegener’s granulomatosis:
Asthma
Recurrent sinusitis
Constitutional sx
Peripheral neuropathy
Mononeuritis multiplex
GI pain
ACR criteria: Patient must have 4 of 6 criteria
- Diagnosis (asthma)
- Eosinophilia (>10 % in peripheral blood)
Elevated Eosinophil count (>10
%)
Elevated ESR,
Elevated CRP
In the presence of clinical suspicion
Initial diagnostic step is a biopsy of an
affected area (skin, renal, muscle, lung,
nerve)
Prednisone alone is sufficient, unless the
symptoms are life threatening
*Higher rate of coronary involvement (aneurysms)*
WAY TO REMEMBER THIS DISEASE LOL:
*If Wegener’s disease and Polyarteritis Nodosa got
together and had a bastard child, it’s name will be
Chung- Strauss, and he’d have asthma*
- Has chronic sinus and nose involvement (usually
doesn’t perforate though)
- Involves the small vessels
- Can cause glomerulonephritis (UA=RBCs++)
- Tends to be ANCA positive (but it’s p-ANCA)
- Granulomatous on biopsy
Like Polyarteritis Nodosa :
- Has neuromuscular involvement
- Can develop mononeuritis multiplex, peripheral
neuropathies
- Postprandial abdominal pain is a common
complaint
Unique to Chang- Straus :
- Patient ALWAYS has asthma or some kind of
reactive airway syndrome
- Patient has eosinophilia (>10 %)
Cough
Arthralgias
Hemoptysis (remember, PAN spares the
lungs >_< )
Hematuria
Purpuric rash
- Paranasal sinusitis
- Mononeuritis multiplex or
polyneuropathy
- Pulmonary infiltrates (on CXR or CT)
- Histological proof of vasculitis
w/extravascular eosinophils
ACD present
UA will generally be abnormal
showing a GMN picture
(RBCs++, RBC casts)
p- ANCA+
Chung- Straus shares features of both Wegener’s
disease and Polyarteritis Nodosa and the patient with Chung- Strauss always
has asthma
- Biopsy shows eosinophils in granuloma
TEMPORAL ARTERITIS (GIANT
CELL ARTERITIS)
Inflammatory disease of the vessels involving the
medium and large arteries of the head/scalp (branches
of the ECA, most prominently the superficial temporal
artery)
Occurs always only in patient over the age of 55
Painful temples
Pain on chewing
Tender scalp
Visual disturbances:
- Diplopia
- Blurry vision
May co-exist with polymyalgia
rheumatica (proximal muscle pain)
PX:
- Scalp tenderness
- Superficial temporal artery is
visibly inflamed/engorged on the
temple or lateral forehead
N/A The best initial diagnostic step is to get an
erythrocyte sedimentation rate (ESR)
Most accurate test: temporal artery biopsy
*If the ESR is elevated, proceed straight to
treatment (Prednisone)*
You must also get a temporal artery biopsy
for confirmation, as it is the most common
test.
Prednisone
TAKAYASU’s ARTERITIS
(PULSELESS DISEASE)
Inflammatory disease of the LARGE VESSELS!!
( implicating the aorta and usually its initial ascending
branches (L Brachiocephalica, R Common Carotid , R
Subclavian a), however, it may descend all the way to
the aortic bifurcation)
Most commonly affects women of childbearing age (
<40 years).
More prevalent in Asians
Variable and non- specific Headache
Malaise
Arthralgia
Visual deficits.
Claudication of the extremities
Stoke
TIA and seizures are rare
manifestations
PX:
- Very important for establishing
suspicion
- Carotid or subclavian bruit
- Aortic regurgitation
- Weak upper extremity pulse
- BP difference between arms of
Diagnosis is likely if pt has 3 of the 6 criteria :
1. Younger than age 40
2. Extremity claudication
3. Decreased pulsation of one or both
brachial arteries
4. Difference in systolic BP of >10 mmHg
between arms
5. Subclavian bruit or abdominal aorta bruit
6. Arteriographic evidence
The most accurate test is arteriography Prednisone is the first line of therapy
Cyclophosphamide or Methotrexate may be added if there is no significant response to
prednisone alone
*Very important to ensure normal BP and manage
other cardiovascular risk factors, especially in
pregnancy*
>10 mmHg
BEÇHET’s SYNDROME Inflammatory, multisystemic disease of SMALL
VESSELS!!
resulting in frequent aneurysm and rupture
Idiopathic
Affects:
- Eyes
- Mucous membranes
- Genitals
*recurrent aphthous ulcers*
Recurrent aphthous ulcers
Painful genital ulcers
Eye pain w/redness (uveitis)
Rash (erythema nodosum acne like
rash)
Rarer but more serious symptom
include:
Meningoencephalitis
GI bleed
Amyloidosis
ISG Criteria for Behcet’s Syndrome:
*MUST HAVE*
- Recurrent oral aphthous ulceration (3x
in 12 month period)
* AND 2 OF THE FOLLOWING*
- Recurrent genital ulceration or scarring
- Eye lesions:
- Anterior uveitis
- Posterior uveitis,
- Retinal vasculitis (dx by ophto)
- Skin lesions:
- Erythema nodosum
- Acneiform nodules
- Papulo-pustular lesions (in patient
who has no other explanation)
BASED ON SYMPTOMS!!!!!
ORAL ULCERS:
(Topical corticosteroids, colchicine)
GENITAL ULCERS:
(Topical corticosteroids)
UVEITIS:
(Topical corticosteroids)
RETINAL VASCULITIS:
Cyclosporine
- Positive patherge test:
- Insert hypodermic needle into
skin, after 48 hours, papule present
in the region
KAWASAKI DISEASE Is also known as mucocutaneous lymph node
Syndrome
Is an acute self limited febrile illness of infants and
children (< 5 yrs)
JAPAN, HAWAII ( ENDEMIC)
KD is a childhood vasculitis that mainly targets
coronary arteries
Coronary artery involvement: – can lead to coronary
thrombosis or aneurysm formation and its rupture
Features:
Oral Erythema
Palmar Erythema
Conjunctivitis
Rash
Desquamation
Edema: FEET & ARMS
Clinical Findings:
- High fever
– Erythematous rash of trunk and
extremities with desquamation of skin.
N/A Neutrophilic leukocytosis
Thrombocytosis :
* characteristic finding*
Elevated ESR
Abnormal ECG
*acute MI*
– Mucosal inflammation : cracked lips,
oral erythema
– Erythema, swelling of hands and feet.
– Localized lymphadenopathy (cervical
adenopathy)
– MCC of an acute MI in children****
BUERGER’s DISEASE Also known as Thromboangitis Obliterans.
Is a peripheral vascular disease of smokers.
Pathology:
Earliest change:
- Acute inflammation involving the small to
medium sized arteries in the extremities (tibial,
popliteal & radial arteries).
Inflammation of vessel:
- Thrombus formation
Clinical Findings:
- Young-middle age, male, heavy
smoker ( Is***l, Japan, India)
Symptoms start between 25 to 40 years:
- Early manifestation:
- Intermittent Claudication
in feet or hands.
(Cramping pain in muscles
after exercise, relieved by
rest)
- Late manifestation:
N/A BIOPSY Early stages of vasculitis frequently cease on discontinuation of smoking.
- Obliterates lumen
- Ischemia
- Gangrene of extremity
Inflammation also extends to adjacent veins and nerves.
(Involvement of entire neurovascular compartment)
- Painful ulcerations of
digits
- Gangrene of the digits
often requiring
amputation.
HENOCH SCHONLEIN PURPURA
(HSP)
A variant of hypersensitivity vasculitis.
Seen in children (MC vasculitis in children)
Rare in adults.
Usually occurs following an upper respiratory infection
Caused by deposition of IgA-C3 immune complexes in
vessel wall.
Vessels involved:
- Arterioles
Clinically characterized by: –
- Palpable purpura over extensor
aspects of arms and legs.
- commonly limited to
lower extremities/
buttocks.
Involvement of:
● GIT
○ colicky abdominal pain,
melena
● Musculoskeletal system:
N/A
Neutrophilic leukocytosis
Deposition of IgA-C3 immune
complexes in:
Skin
Renal lesions
STEROIDS
- Capillaries and venules of:
- Skin
- GIT
- Kidney
- Musculoskeletal system.
○ Arthralgia (non migratory)
○ Myalgias
● Kidneys:
○ hematuria due to focal
proliferative GN
● Lung:
○ Rare
INFECTIOUS VASCULITIS Fungal vasculitis: vessel invading fungi
- Mucor
- Aspergillus
- Candida
Rocky Mountain spotted fever:
- Rickettsia
Disseminated meningococcemia:
- Small vessel vasculitis
N/A
- petechial hemorrhages
Infective endocarditis:
- Roth’s spots in retina
- Janeway’s lesions on hands (painless)
- Osler’s nodes on hands (painful)
- Glomerulonephritis
SJÖGREN’s SYNDROME Autoimmune, inflammatory and destructive disease of
the exocrine organs (primarily parotid glands, lacrimal
ducts, etc)
Affects 0.1- 4 % of the US population
Female is preponderance
Primary and secondary forms
History of numerous dental caries, due to
chronically dry mouth
In many cases, the patients will have a history of
another autoimmune disease (SLE, RA, etc) as
many as 1/3 of RA patients also have Sjogren’s
syndrome
The major symptoms are:
- Dry eyes (xerophthalmia)
- Dry mouth (xerostomia)
- Parotid gland enlargement
Typical presentation :
- dry eyes / dry mouth in female
patients
Pts must have 2 of the following 3:
Ocular staining score at least 3
Biopsy of salivary gland (single most accurate
test) showing lymphocytic sialadenitis
Positive ANA . Specifically Anti-Ro and Anti-La
CRITERIA BASED
*Biopsy of salivary gland (single most
accurate test*
Schirmer’s test:
- This test consists of placing a small
Lymphoma is a serious complication of SS, occurring LATE in the disease.
Risk of lymphoma is 44 times greater than the general population
20-30% of patients have vasculitis.
Renal involvement in 10% of patients interstitial nephritis
Complications:
- NonHodgkin’s lymphoma
- Neonatal lupus
- Some degree of parotid gland
swelling may be present
Mild to moderate sensorineural hearing
loss in high frequencies.
*Anti-cardiolipin antibodies has been
demonstrated, but no cochlear or
vestibular pathology has been found*
Nodes:
- Granulomatous and
non-granulomatous laryngeal
nodes
- Bamboo nodes - whitish or
yellowish transverse submucosal
lesions localized to middle third
of vocal fold.
Rare first symptom:
- Hoarseness
Lacrimal-gland biopsy
- Focal lymphocytic sialadenitis in minor
salivary glands containing more than 50
lymphocytes per 4 mm of glandular tissue
strip of filter paper inside the lower
eyelid (conjunctiva sac).
- The eyes are closed for 5 minutes.
- The paper is then removed and the
amount of moisture is measured.
- A young person normally moistens 15
mm of each paper strip.
- Because hyperlacrimation occurs
with aging, 33% of normal elderly
persons may wet only 10 mm in 5
minutes
- Persons with Sjögren's syndrome
moisten less than 5 mm in 5 minutes
RESULTS OF SCHIRMER TEST!!!!
1. Normal:
- which is ≥15 mm wetting of the paper
after 5 minutes.
Renal tubular acidosis also seen
Thyroid abnormalities:
- “Hashimotos’s thyroiditis” can
be found in up to 1/2 of Sjögren’s
syndrome patients
- 10-15% of Primary Sjögren’s
syndrome are clinically
hypothyroid
- Therefore, all patients with this
syndrome should undergo
routine evaluation of thyroid
function
50% having nasal mucosa hypertrophy
13% have septal perforations.
Pulmonary Symptoms:
- Xerotrachea (dry cough)
- Dyspnea
2. Mild:
- which is 14-9 mm wetting of the
paper after 5 minutes
3. Moderate:
- which is 8-4 mm wetting of the paper
after 5 minutes.
4. Severe:
- which is < 4 mm wetting of the paper
after 5 minutes.
Esophageal spasm and dysmotility are
also seen, but may be secondary to
absence of saliva,
- Atrophic gastritis
- celiac disease
GOUT -- Crystal deposition in the joint---
- inflammation, pain
-- Acute pain attacks
-- Men affected more than women
-- Blacks more than whites.
-- Average age of initial onset is 30-60 years
*Most often in early patients*
-- Urate crystal deposition in the joint, resulting in
tends to occur in older men( RF= age):
increased risk in:
- Overweight
- Alcohol drinkers
- Patients on diuretics (loop ( furosemide)
and thiazide)
Excruciating monoarticular joint pain
*may wake up the patient*
Overlying erythema
visible swelling
occasionally there may be nodules in
the soft tissue.
*Inflamed joints*
Fever may be present if there is
“polyarticular” involvement
N/A The best initial diagnostic step is: synovial
fluid analysis (arthrocentesis)
*don’t need to do every time*
Maybe diagnosing clinically if the patient
has had several confirmed attacks of gout
and this episode is similar
The best initial step in therapy is NSAIDs
*We treat the pain first !*
To prevent further gout attacks, patients should be started on:
- Allopurinol:
- Usual dose is 300 mg/day. The Maximum recommended dose is 800 mg/day.
- Dose depends on Kidney function:
- GFR
- Check creatinine
- In renal insufficiency dose should be decreased to 200 mg/day for
creatinine clearance < 60ml/min and to 100 mg/day if clearance < 30
ml/min).
- Start with small doses of allopurinol to reduce the risk of precipitating an
acute gout attack.
- Most common side effects are:
Maybe mistaken for cellulitis (over the affected joints)
Hallmark: Only in Gout Patients
inflammation and pain.
-- Most commonly first occurs in the big toe
(significant pain will occur too)
● May also occur in the:
○ Ankle
○ Knee
○ Proximal Interphalangeals (PIPs)
○ Distal Interphalangeals (DIPs)
○ Maybe polyarticular (fever also)
*monoarticular = no fever*
- Rash (2% of patients)
- rarely patients can develop exfoliative dermatitis that can be lethal
Colchicine may be used as well, but it has “significant GI effects”
Febuxostat is another, more recent drug on the market
Patients with multiple gout attacks should undergo:
- A 24- hour uric acid collection
*Do not fall for the trap of ordering serum uric acid levels !!! They don’t
correspond to gout attacks!*
PSEUDOGOUT Deposition of calcium pyrophosphate dehydrate,
calcium oxalate or calcium hydroxyapatite crystals in
the joint, resulting inflammation and pain
Tends to occur in older patients with pre-
existing joint disease and in patients with
metabolic or electrolyte disorders (particularly
involving calcium) *Hypercalcemia,
Hypophosphatemia, Hyperparathyroidism,
Hemochromatosis, Hypomagnesemia*
Presentation is almost identical to gout
not GOAT ( lol >_<)
Most common joint affected is the
knee.
The wrist, ankle, and shoulder may also
get affected
N/A Uric Acid: normal values
ranges:
- from 4.0 to 8.6 mg/dl in
men
- from 3.0 to 5.9 mg/dl in
women.
Urinary levels are normal
below 750 mg/24h
Urinary levels above 750 mg/dl
in 24h in gout or > 1100 mg/dl
in asymptomatic
hyperuricemia indicates urate
overproduction
The best diagnostic test is: Synovial fluid
analysis (arthrocentesis)
Like gout, the best initial therapy is NSAIDs
*Consider workup for underlying cause*
If Comprehensive Metabolic Panel (CMP) shows elevated calcium levels, consider getting a
PTH level
May put patient on low dose colchicine as a prophylactic for future attacks.
Diet is usually impractical, ineffective and rarely adhered to in clinical practice ( for
both )
SCLERODERMA
(CREST SYNDROME & SYSTEMIC
SCLEROSIS)
Scleroderma is a symptom (thickening of the
skin)
A term that can refer to two similar but separate
clinical syndromes
Almost all (more than 90%) of patients
with scleroderma also have Raynaud's
phenomenon.
Systemic Sclerosis:
ACR criteria for Systemic sclerosis
Patients must have one major criteria or two
minor criteria
CREST SYNDROME:
Clinical, major differential is systemic
sclerosis
Systemic Sclerosis:
- Symptomatic.
Major focus is in preventing scleroderma renal crisis (malignant
hypertension) and minimizing pulmonary hypertension
Pulmonary manifestations of systemic sclerosis occur in more than 70% of
Systemic sclerosis *aka diffuse systemic sclerosis* →
CREST + MULTI ORGAN
Systemic sclerosis and CREST syndrome both involve
cutaneous symptoms (skin thickening, calcinosis,
Raynaud syndrome, histological changes)
“Systemic Sclerosis” can affect the:
- Kidneys (renal insufficiency)
- The heart (CHF)
- Lungs (pulmonary fibrosis)
*CREST SYNDROME DOES NOT AFFECT*
Systemic sclerosis is:
- CREST syndrome PLUS Multiple internal organ
involvement
Idiopathic and Female predominance
Can have many different presentations
CREST syndrome *aka limited systemic sclerosis,
cutaneous sclerosis*
- Generally have symptoms similar
to CREST syndrome
- is not limited to peripheral
extremities and often affects the
face and neck.
- Other non- specific symptoms
may be present:
- Palpitations
- Shortness of breath
- Fatigue
- Weakness
Major criteria:
- Proximal scleroderma
Minor criteria:
- Sclerodactyly
- Digital pitting scars or loss of substance
from finger pad
- Basilar pulmonary fibrosis
Patients with CREST syndrome should have
titers ran for ANA (90% of patients will be+)
and anti- Scl- 70
Systemic Sclerosis:
Clinical, supported with laboratory evidence
Patients + for Anti- Scl-70 are more likely to
have diffuse systemic sclerosis
Treating CREST and Systemic sclerosis:
- Focused on symptoms and in systemic sclerosis, preventing scleroderma renal
crisis, and limiting any other complications
- Skin thickening : D-penicillamine
- Raynaud attacks: Calcium channel blockers (nifedipine) *sometimes a-blockers &
Nitrates*
- Scleroderma renal crisis (ACE inhibitors)
- GI symptoms: Proton- pump inhibitors manage closely for Barrett’s esophagus
PULMONARY:
- Most common SSc-related cause of death.
- Pulmonary diffuse fibrosis is a common cause of severe restrictive lung disease
- CXR shows interstitial thickening, most easily seen in the lower lung fields.
- ECG→ Right ventricular hypertrophy
*Some patients with interstitial lung
disease develop slowly progressive
patients
In Diffuse systemic sclerosis:
- Interstitial lung disease (ILD) more common
In Limited systemic sclerosis:
- Pulmonary hypertension is more common
The pulmonic component of the second heart sound is accentuated, and
ultimately, right-sided cardiac failure develops.
Diagnosis is confirmed by:
- echocardiogram or by right heart catheterization.
res
Sig
(PA
15%
cat
RE
CA
*Ac
Car
Sec
Or
MU
Mo
Als
NE
RHEUMATOLOGY DISEASES TABLE ALL IN ONE..

More Related Content

Similar to RHEUMATOLOGY DISEASES TABLE ALL IN ONE..

Rheumatoid arthritis
Rheumatoid arthritisRheumatoid arthritis
Rheumatoid arthritisKiran Bikkad
 
ANTERIOR TIBIAL SYNDROME AND REYNAUD''S DISEASE
ANTERIOR TIBIAL SYNDROME  AND REYNAUD''S DISEASEANTERIOR TIBIAL SYNDROME  AND REYNAUD''S DISEASE
ANTERIOR TIBIAL SYNDROME AND REYNAUD''S DISEASEDr Thouseef Abdul Majeed
 
Rheumatoid Arthritis
Rheumatoid ArthritisRheumatoid Arthritis
Rheumatoid Arthritisazamcmc50
 
Inflamatory arthritis
Inflamatory arthritisInflamatory arthritis
Inflamatory arthritisdrangelosmith
 
Approach to a child with arthritis by dr praman kushwah
Approach to a child with arthritis by dr praman kushwahApproach to a child with arthritis by dr praman kushwah
Approach to a child with arthritis by dr praman kushwahDr Praman Kushwah
 
Rheumatoid Arthritis for Medical and Pharmacy Students
Rheumatoid Arthritis for Medical and Pharmacy StudentsRheumatoid Arthritis for Medical and Pharmacy Students
Rheumatoid Arthritis for Medical and Pharmacy Studentsarun chand roby
 
Athritis (RA, OA, Gout) MSN.pptx
Athritis (RA, OA, Gout) MSN.pptxAthritis (RA, OA, Gout) MSN.pptx
Athritis (RA, OA, Gout) MSN.pptxdakshkarwal
 
Final rheumatoid arthritis
Final rheumatoid arthritisFinal rheumatoid arthritis
Final rheumatoid arthritisAmer
 
Final rheumatoid arthritis
Final rheumatoid arthritisFinal rheumatoid arthritis
Final rheumatoid arthritisAmer
 
Complex regional pain syndrome Petrus Iitula
Complex regional pain syndrome   Petrus IitulaComplex regional pain syndrome   Petrus Iitula
Complex regional pain syndrome Petrus IitulaPetrus Iitula
 
Seronegative Arthropathy.pptx
Seronegative  Arthropathy.pptxSeronegative  Arthropathy.pptx
Seronegative Arthropathy.pptxJoydeep Tripathi
 
4. Peripheral-Neuropathy.ppt
4. Peripheral-Neuropathy.ppt4. Peripheral-Neuropathy.ppt
4. Peripheral-Neuropathy.pptAbwoneKenneth
 
final_rheumatoid_arthritis.ppt
final_rheumatoid_arthritis.pptfinal_rheumatoid_arthritis.ppt
final_rheumatoid_arthritis.pptaartichande
 
final_rheumatoid_arthritis.ppt
final_rheumatoid_arthritis.pptfinal_rheumatoid_arthritis.ppt
final_rheumatoid_arthritis.pptneeti70
 
final rheumatoid arthritis medical .ppt
final rheumatoid arthritis medical  .pptfinal rheumatoid arthritis medical  .ppt
final rheumatoid arthritis medical .pptShivani Bhardwaj
 

Similar to RHEUMATOLOGY DISEASES TABLE ALL IN ONE.. (20)

Rheumatoid arthritis
Rheumatoid arthritisRheumatoid arthritis
Rheumatoid arthritis
 
ANTERIOR TIBIAL SYNDROME AND REYNAUD''S DISEASE
ANTERIOR TIBIAL SYNDROME  AND REYNAUD''S DISEASEANTERIOR TIBIAL SYNDROME  AND REYNAUD''S DISEASE
ANTERIOR TIBIAL SYNDROME AND REYNAUD''S DISEASE
 
koch's spine
koch's spinekoch's spine
koch's spine
 
Rheumatoid Arthritis
Rheumatoid ArthritisRheumatoid Arthritis
Rheumatoid Arthritis
 
Rheumatology 2
Rheumatology 2Rheumatology 2
Rheumatology 2
 
Spondyloarthropathy
SpondyloarthropathySpondyloarthropathy
Spondyloarthropathy
 
Inflamatory arthritis
Inflamatory arthritisInflamatory arthritis
Inflamatory arthritis
 
Tuberculosis
TuberculosisTuberculosis
Tuberculosis
 
Approach to a child with arthritis by dr praman kushwah
Approach to a child with arthritis by dr praman kushwahApproach to a child with arthritis by dr praman kushwah
Approach to a child with arthritis by dr praman kushwah
 
Rheumatoid Arthritis for Medical and Pharmacy Students
Rheumatoid Arthritis for Medical and Pharmacy StudentsRheumatoid Arthritis for Medical and Pharmacy Students
Rheumatoid Arthritis for Medical and Pharmacy Students
 
juvenile Arthritis
juvenile Arthritis juvenile Arthritis
juvenile Arthritis
 
Athritis (RA, OA, Gout) MSN.pptx
Athritis (RA, OA, Gout) MSN.pptxAthritis (RA, OA, Gout) MSN.pptx
Athritis (RA, OA, Gout) MSN.pptx
 
Final rheumatoid arthritis
Final rheumatoid arthritisFinal rheumatoid arthritis
Final rheumatoid arthritis
 
Final rheumatoid arthritis
Final rheumatoid arthritisFinal rheumatoid arthritis
Final rheumatoid arthritis
 
Complex regional pain syndrome Petrus Iitula
Complex regional pain syndrome   Petrus IitulaComplex regional pain syndrome   Petrus Iitula
Complex regional pain syndrome Petrus Iitula
 
Seronegative Arthropathy.pptx
Seronegative  Arthropathy.pptxSeronegative  Arthropathy.pptx
Seronegative Arthropathy.pptx
 
4. Peripheral-Neuropathy.ppt
4. Peripheral-Neuropathy.ppt4. Peripheral-Neuropathy.ppt
4. Peripheral-Neuropathy.ppt
 
final_rheumatoid_arthritis.ppt
final_rheumatoid_arthritis.pptfinal_rheumatoid_arthritis.ppt
final_rheumatoid_arthritis.ppt
 
final_rheumatoid_arthritis.ppt
final_rheumatoid_arthritis.pptfinal_rheumatoid_arthritis.ppt
final_rheumatoid_arthritis.ppt
 
final rheumatoid arthritis medical .ppt
final rheumatoid arthritis medical  .pptfinal rheumatoid arthritis medical  .ppt
final rheumatoid arthritis medical .ppt
 

Recently uploaded

Call Girls Kochi Just Call 9907093804 Top Class Call Girl Service Available
Call Girls Kochi Just Call 9907093804 Top Class Call Girl Service AvailableCall Girls Kochi Just Call 9907093804 Top Class Call Girl Service Available
Call Girls Kochi Just Call 9907093804 Top Class Call Girl Service AvailableDipal Arora
 
Call Girls Tirupati Just Call 9907093804 Top Class Call Girl Service Available
Call Girls Tirupati Just Call 9907093804 Top Class Call Girl Service AvailableCall Girls Tirupati Just Call 9907093804 Top Class Call Girl Service Available
Call Girls Tirupati Just Call 9907093804 Top Class Call Girl Service AvailableDipal Arora
 
Top Rated Bangalore Call Girls Richmond Circle ⟟ 9332606886 ⟟ Call Me For Ge...
Top Rated Bangalore Call Girls Richmond Circle ⟟  9332606886 ⟟ Call Me For Ge...Top Rated Bangalore Call Girls Richmond Circle ⟟  9332606886 ⟟ Call Me For Ge...
Top Rated Bangalore Call Girls Richmond Circle ⟟ 9332606886 ⟟ Call Me For Ge...narwatsonia7
 
Call Girls Faridabad Just Call 9907093804 Top Class Call Girl Service Available
Call Girls Faridabad Just Call 9907093804 Top Class Call Girl Service AvailableCall Girls Faridabad Just Call 9907093804 Top Class Call Girl Service Available
Call Girls Faridabad Just Call 9907093804 Top Class Call Girl Service AvailableDipal Arora
 
VIP Call Girls Indore Kirti 💚😋 9256729539 🚀 Indore Escorts
VIP Call Girls Indore Kirti 💚😋  9256729539 🚀 Indore EscortsVIP Call Girls Indore Kirti 💚😋  9256729539 🚀 Indore Escorts
VIP Call Girls Indore Kirti 💚😋 9256729539 🚀 Indore Escortsaditipandeya
 
Top Quality Call Girl Service Kalyanpur 6378878445 Available Call Girls Any Time
Top Quality Call Girl Service Kalyanpur 6378878445 Available Call Girls Any TimeTop Quality Call Girl Service Kalyanpur 6378878445 Available Call Girls Any Time
Top Quality Call Girl Service Kalyanpur 6378878445 Available Call Girls Any TimeCall Girls Delhi
 
Best Rate (Guwahati ) Call Girls Guwahati ⟟ 8617370543 ⟟ High Class Call Girl...
Best Rate (Guwahati ) Call Girls Guwahati ⟟ 8617370543 ⟟ High Class Call Girl...Best Rate (Guwahati ) Call Girls Guwahati ⟟ 8617370543 ⟟ High Class Call Girl...
Best Rate (Guwahati ) Call Girls Guwahati ⟟ 8617370543 ⟟ High Class Call Girl...Dipal Arora
 
Lucknow Call girls - 8800925952 - 24x7 service with hotel room
Lucknow Call girls - 8800925952 - 24x7 service with hotel roomLucknow Call girls - 8800925952 - 24x7 service with hotel room
Lucknow Call girls - 8800925952 - 24x7 service with hotel roomdiscovermytutordmt
 
College Call Girls in Haridwar 9667172968 Short 4000 Night 10000 Best call gi...
College Call Girls in Haridwar 9667172968 Short 4000 Night 10000 Best call gi...College Call Girls in Haridwar 9667172968 Short 4000 Night 10000 Best call gi...
College Call Girls in Haridwar 9667172968 Short 4000 Night 10000 Best call gi...perfect solution
 
Call Girls Gwalior Just Call 8617370543 Top Class Call Girl Service Available
Call Girls Gwalior Just Call 8617370543 Top Class Call Girl Service AvailableCall Girls Gwalior Just Call 8617370543 Top Class Call Girl Service Available
Call Girls Gwalior Just Call 8617370543 Top Class Call Girl Service AvailableDipal Arora
 
Call Girls Dehradun Just Call 9907093804 Top Class Call Girl Service Available
Call Girls Dehradun Just Call 9907093804 Top Class Call Girl Service AvailableCall Girls Dehradun Just Call 9907093804 Top Class Call Girl Service Available
Call Girls Dehradun Just Call 9907093804 Top Class Call Girl Service AvailableDipal Arora
 
💎VVIP Kolkata Call Girls Parganas🩱7001035870🩱Independent Girl ( Ac Rooms Avai...
💎VVIP Kolkata Call Girls Parganas🩱7001035870🩱Independent Girl ( Ac Rooms Avai...💎VVIP Kolkata Call Girls Parganas🩱7001035870🩱Independent Girl ( Ac Rooms Avai...
💎VVIP Kolkata Call Girls Parganas🩱7001035870🩱Independent Girl ( Ac Rooms Avai...Taniya Sharma
 
Premium Call Girls Cottonpet Whatsapp 7001035870 Independent Escort Service
Premium Call Girls Cottonpet Whatsapp 7001035870 Independent Escort ServicePremium Call Girls Cottonpet Whatsapp 7001035870 Independent Escort Service
Premium Call Girls Cottonpet Whatsapp 7001035870 Independent Escort Servicevidya singh
 
Manyata Tech Park ( Call Girls ) Bangalore ✔ 6297143586 ✔ Hot Model With Sexy...
Manyata Tech Park ( Call Girls ) Bangalore ✔ 6297143586 ✔ Hot Model With Sexy...Manyata Tech Park ( Call Girls ) Bangalore ✔ 6297143586 ✔ Hot Model With Sexy...
Manyata Tech Park ( Call Girls ) Bangalore ✔ 6297143586 ✔ Hot Model With Sexy...vidya singh
 
(Low Rate RASHMI ) Rate Of Call Girls Jaipur ❣ 8445551418 ❣ Elite Models & Ce...
(Low Rate RASHMI ) Rate Of Call Girls Jaipur ❣ 8445551418 ❣ Elite Models & Ce...(Low Rate RASHMI ) Rate Of Call Girls Jaipur ❣ 8445551418 ❣ Elite Models & Ce...
(Low Rate RASHMI ) Rate Of Call Girls Jaipur ❣ 8445551418 ❣ Elite Models & Ce...parulsinha
 
Call Girls Visakhapatnam Just Call 9907093804 Top Class Call Girl Service Ava...
Call Girls Visakhapatnam Just Call 9907093804 Top Class Call Girl Service Ava...Call Girls Visakhapatnam Just Call 9907093804 Top Class Call Girl Service Ava...
Call Girls Visakhapatnam Just Call 9907093804 Top Class Call Girl Service Ava...Dipal Arora
 
VIP Service Call Girls Sindhi Colony 📳 7877925207 For 18+ VIP Call Girl At Th...
VIP Service Call Girls Sindhi Colony 📳 7877925207 For 18+ VIP Call Girl At Th...VIP Service Call Girls Sindhi Colony 📳 7877925207 For 18+ VIP Call Girl At Th...
VIP Service Call Girls Sindhi Colony 📳 7877925207 For 18+ VIP Call Girl At Th...jageshsingh5554
 
Top Rated Bangalore Call Girls Ramamurthy Nagar ⟟ 9332606886 ⟟ Call Me For G...
Top Rated Bangalore Call Girls Ramamurthy Nagar ⟟  9332606886 ⟟ Call Me For G...Top Rated Bangalore Call Girls Ramamurthy Nagar ⟟  9332606886 ⟟ Call Me For G...
Top Rated Bangalore Call Girls Ramamurthy Nagar ⟟ 9332606886 ⟟ Call Me For G...narwatsonia7
 
Pondicherry Call Girls Book Now 9630942363 Top Class Pondicherry Escort Servi...
Pondicherry Call Girls Book Now 9630942363 Top Class Pondicherry Escort Servi...Pondicherry Call Girls Book Now 9630942363 Top Class Pondicherry Escort Servi...
Pondicherry Call Girls Book Now 9630942363 Top Class Pondicherry Escort Servi...Genuine Call Girls
 
(👑VVIP ISHAAN ) Russian Call Girls Service Navi Mumbai🖕9920874524🖕Independent...
(👑VVIP ISHAAN ) Russian Call Girls Service Navi Mumbai🖕9920874524🖕Independent...(👑VVIP ISHAAN ) Russian Call Girls Service Navi Mumbai🖕9920874524🖕Independent...
(👑VVIP ISHAAN ) Russian Call Girls Service Navi Mumbai🖕9920874524🖕Independent...Taniya Sharma
 

Recently uploaded (20)

Call Girls Kochi Just Call 9907093804 Top Class Call Girl Service Available
Call Girls Kochi Just Call 9907093804 Top Class Call Girl Service AvailableCall Girls Kochi Just Call 9907093804 Top Class Call Girl Service Available
Call Girls Kochi Just Call 9907093804 Top Class Call Girl Service Available
 
Call Girls Tirupati Just Call 9907093804 Top Class Call Girl Service Available
Call Girls Tirupati Just Call 9907093804 Top Class Call Girl Service AvailableCall Girls Tirupati Just Call 9907093804 Top Class Call Girl Service Available
Call Girls Tirupati Just Call 9907093804 Top Class Call Girl Service Available
 
Top Rated Bangalore Call Girls Richmond Circle ⟟ 9332606886 ⟟ Call Me For Ge...
Top Rated Bangalore Call Girls Richmond Circle ⟟  9332606886 ⟟ Call Me For Ge...Top Rated Bangalore Call Girls Richmond Circle ⟟  9332606886 ⟟ Call Me For Ge...
Top Rated Bangalore Call Girls Richmond Circle ⟟ 9332606886 ⟟ Call Me For Ge...
 
Call Girls Faridabad Just Call 9907093804 Top Class Call Girl Service Available
Call Girls Faridabad Just Call 9907093804 Top Class Call Girl Service AvailableCall Girls Faridabad Just Call 9907093804 Top Class Call Girl Service Available
Call Girls Faridabad Just Call 9907093804 Top Class Call Girl Service Available
 
VIP Call Girls Indore Kirti 💚😋 9256729539 🚀 Indore Escorts
VIP Call Girls Indore Kirti 💚😋  9256729539 🚀 Indore EscortsVIP Call Girls Indore Kirti 💚😋  9256729539 🚀 Indore Escorts
VIP Call Girls Indore Kirti 💚😋 9256729539 🚀 Indore Escorts
 
Top Quality Call Girl Service Kalyanpur 6378878445 Available Call Girls Any Time
Top Quality Call Girl Service Kalyanpur 6378878445 Available Call Girls Any TimeTop Quality Call Girl Service Kalyanpur 6378878445 Available Call Girls Any Time
Top Quality Call Girl Service Kalyanpur 6378878445 Available Call Girls Any Time
 
Best Rate (Guwahati ) Call Girls Guwahati ⟟ 8617370543 ⟟ High Class Call Girl...
Best Rate (Guwahati ) Call Girls Guwahati ⟟ 8617370543 ⟟ High Class Call Girl...Best Rate (Guwahati ) Call Girls Guwahati ⟟ 8617370543 ⟟ High Class Call Girl...
Best Rate (Guwahati ) Call Girls Guwahati ⟟ 8617370543 ⟟ High Class Call Girl...
 
Lucknow Call girls - 8800925952 - 24x7 service with hotel room
Lucknow Call girls - 8800925952 - 24x7 service with hotel roomLucknow Call girls - 8800925952 - 24x7 service with hotel room
Lucknow Call girls - 8800925952 - 24x7 service with hotel room
 
College Call Girls in Haridwar 9667172968 Short 4000 Night 10000 Best call gi...
College Call Girls in Haridwar 9667172968 Short 4000 Night 10000 Best call gi...College Call Girls in Haridwar 9667172968 Short 4000 Night 10000 Best call gi...
College Call Girls in Haridwar 9667172968 Short 4000 Night 10000 Best call gi...
 
Call Girls Gwalior Just Call 8617370543 Top Class Call Girl Service Available
Call Girls Gwalior Just Call 8617370543 Top Class Call Girl Service AvailableCall Girls Gwalior Just Call 8617370543 Top Class Call Girl Service Available
Call Girls Gwalior Just Call 8617370543 Top Class Call Girl Service Available
 
Call Girls Dehradun Just Call 9907093804 Top Class Call Girl Service Available
Call Girls Dehradun Just Call 9907093804 Top Class Call Girl Service AvailableCall Girls Dehradun Just Call 9907093804 Top Class Call Girl Service Available
Call Girls Dehradun Just Call 9907093804 Top Class Call Girl Service Available
 
💎VVIP Kolkata Call Girls Parganas🩱7001035870🩱Independent Girl ( Ac Rooms Avai...
💎VVIP Kolkata Call Girls Parganas🩱7001035870🩱Independent Girl ( Ac Rooms Avai...💎VVIP Kolkata Call Girls Parganas🩱7001035870🩱Independent Girl ( Ac Rooms Avai...
💎VVIP Kolkata Call Girls Parganas🩱7001035870🩱Independent Girl ( Ac Rooms Avai...
 
Premium Call Girls Cottonpet Whatsapp 7001035870 Independent Escort Service
Premium Call Girls Cottonpet Whatsapp 7001035870 Independent Escort ServicePremium Call Girls Cottonpet Whatsapp 7001035870 Independent Escort Service
Premium Call Girls Cottonpet Whatsapp 7001035870 Independent Escort Service
 
Manyata Tech Park ( Call Girls ) Bangalore ✔ 6297143586 ✔ Hot Model With Sexy...
Manyata Tech Park ( Call Girls ) Bangalore ✔ 6297143586 ✔ Hot Model With Sexy...Manyata Tech Park ( Call Girls ) Bangalore ✔ 6297143586 ✔ Hot Model With Sexy...
Manyata Tech Park ( Call Girls ) Bangalore ✔ 6297143586 ✔ Hot Model With Sexy...
 
(Low Rate RASHMI ) Rate Of Call Girls Jaipur ❣ 8445551418 ❣ Elite Models & Ce...
(Low Rate RASHMI ) Rate Of Call Girls Jaipur ❣ 8445551418 ❣ Elite Models & Ce...(Low Rate RASHMI ) Rate Of Call Girls Jaipur ❣ 8445551418 ❣ Elite Models & Ce...
(Low Rate RASHMI ) Rate Of Call Girls Jaipur ❣ 8445551418 ❣ Elite Models & Ce...
 
Call Girls Visakhapatnam Just Call 9907093804 Top Class Call Girl Service Ava...
Call Girls Visakhapatnam Just Call 9907093804 Top Class Call Girl Service Ava...Call Girls Visakhapatnam Just Call 9907093804 Top Class Call Girl Service Ava...
Call Girls Visakhapatnam Just Call 9907093804 Top Class Call Girl Service Ava...
 
VIP Service Call Girls Sindhi Colony 📳 7877925207 For 18+ VIP Call Girl At Th...
VIP Service Call Girls Sindhi Colony 📳 7877925207 For 18+ VIP Call Girl At Th...VIP Service Call Girls Sindhi Colony 📳 7877925207 For 18+ VIP Call Girl At Th...
VIP Service Call Girls Sindhi Colony 📳 7877925207 For 18+ VIP Call Girl At Th...
 
Top Rated Bangalore Call Girls Ramamurthy Nagar ⟟ 9332606886 ⟟ Call Me For G...
Top Rated Bangalore Call Girls Ramamurthy Nagar ⟟  9332606886 ⟟ Call Me For G...Top Rated Bangalore Call Girls Ramamurthy Nagar ⟟  9332606886 ⟟ Call Me For G...
Top Rated Bangalore Call Girls Ramamurthy Nagar ⟟ 9332606886 ⟟ Call Me For G...
 
Pondicherry Call Girls Book Now 9630942363 Top Class Pondicherry Escort Servi...
Pondicherry Call Girls Book Now 9630942363 Top Class Pondicherry Escort Servi...Pondicherry Call Girls Book Now 9630942363 Top Class Pondicherry Escort Servi...
Pondicherry Call Girls Book Now 9630942363 Top Class Pondicherry Escort Servi...
 
(👑VVIP ISHAAN ) Russian Call Girls Service Navi Mumbai🖕9920874524🖕Independent...
(👑VVIP ISHAAN ) Russian Call Girls Service Navi Mumbai🖕9920874524🖕Independent...(👑VVIP ISHAAN ) Russian Call Girls Service Navi Mumbai🖕9920874524🖕Independent...
(👑VVIP ISHAAN ) Russian Call Girls Service Navi Mumbai🖕9920874524🖕Independent...
 

RHEUMATOLOGY DISEASES TABLE ALL IN ONE..

  • 1. DISEASE: WHAT IS IT? HISTORY SYMPTOMS / PX CRITERIA (IF THERE IS) LABS DIAGNOSIS (INITIAL /ACCURATE) TREATMENT EXTRA NOTES!!!
  • 2. SERONEGATIVE SPONDYLOARTHROPATHIES (Ankylosing spondylitis Psoriatic arthritis Reactive arthritis Enteropathic arthropathy) Ankylosing spondylitis: - Inflammatory disorder of the axial skeleton and the peripheral joints (Shoulder, hip) - Distinct from most autoimmune disease in that it is predominant in young men who are “under the age of 40” - MEN affected more than women by about 4:1 Reactive Arthritis: - Inflammatory arthritis occurring after or during a bacterial infection - Two types which presents similarly: - Post Diarrheal: - occurs after infection with: - Campylobacter - Salmonella - Shigella - Reiter’s syndrome: - occurs during or after: - - nongonococcal urethritis Ankylosing spondylitis: N/A Reactive Arthritis History will be very important: for instance, arthritis (>1 month) + urethritis has almost 99 % specificity for Reiter’s syndrome Psoriatic Arthritis (PsA): Ankylosing spondylitis: - The primary complaint tends to be persistent and chronic refractory lower back pain * worst in the morning and improves throughout the day (like most autoimmune arthropathies)* PX: - On examination patients will have a loss of lumbar lordosis which indicates the characteristic fusion of lumbar spine processes (this may make the patient susceptible to spine fracture) Reactive Arthritis: - Primarily affects: - weight bearing joints (knees, hips, lower back) - Uveitis Ankylosing spondylitis: N/A Reactive Arthritis: N/A Psoriatic Arthritis: Classification criteria for psoriatic arthritis (CASPAR): If the patient has 3 or more points there is > 90 % sensitivity and almost 99 % specificity for psoriatic arthritis 1. Family history of psoriasis: a. Current (2 points) b. Previous (1 point) 2. Psoriatic nail changes (1 point) Ankylosing spondylitis: N/A Reactive Arthritis: N/A Psoriatic Arthritis (PsA): N/A Enteropathic Arthritis: N/A Ankylosing spondylitis: The best first step is: based on clinical symptom and on lumbar spinal XR Schober’s test: - A useful clinical tool for detecting a loss of lumbar lordosis and therefore possibly ankylosing spondylitis - An increase of < 5 cm is considered a positive Schober’s test and points to a pathologic process Reactive Arthritis: - Can be made clinically. - Should attempt to find the source of the infection - Patients with a history of diarrhea within the last 2-6 weeks can be considered to have: Ankylosing spondylitis: The best first step in therapy of patients with AS is: - NSAIDs (Indomethacin; Ibuprofen) - Physical therapy. SECOND LINE AGENT: - TNF inhibitors (infliximab, adalimumab, etanercept) SURGERY: - Hip replacement surgery - Lumbar Spinal Osteotomy Reactive Arthritis: - NSAIDS are the cornerstone of therapy - *Reactive arthritis and Reiter’s syndrome are self- limited* Reiter’s Syndrome: - a 3 months course of tetracycline may be of benefit Derm symptoms: - Topical corticosteroids (betamethasone) Uveitis: - Optho consultation All seronegative spondyloarthropathies share one thing in common: they all share the presence of HLA B-27 antigen (from the mutation on chromosome 6)
  • 3. (Chlamydia trachomatis) Psoriatic Arthritis (PsA): - Inflammatory, erosive arthritis associated with the skin condition psoriasis Risk Factors: - Personal and Family history of psoriasis. - White > blacks, Hispanics - MOA is around 35-50 years Enteropathic Arthritis: - Arthritis that is accompanied by and particularly associated with flares of IBD - About one in five people with Crohn's or ulcerative colitis will develop enteropathic arthritis N/A Enteropathic Arthritis: N/A - Dactylitis - Low back pain - Circinate balanitis - Small, hard nodules on soles of feet - Possible continued symptom of infection (diarrhea, dysuria) PX: 1. Knee joint may be: a. Warm b. Red c. fluctuant 2. In that case the best initial step is to get a: a. synovial analysis 3. Synovial fluid will be high for WBCs,negative culture Psoriatic Arthritis (PsA): 3. Dactylitis (1 point) 4. Radiographic changes (1 point) : a. Osteophytes b. joint erosion 5. Negative RF (1 point) POOR PROGNOSIS: 1. Extensive skin involvement 2. A strong family history of psoriasis 3. Female gender 4. Disease onset at < 20 years of age 5. Expression of HLA-B27, -DR3 or -DR4 alleles 6. Polyarticular or erosive disease Enteropathic Arthritis: - a post diarrheal etiology - Patient without a history of diarrhea with a questionable sexual history should consider a : - throat and urogenital culture/ - PCR for Chlamydia Psoriatic Arthritis: Criteria based Enteropathic Arthritis: N/A - Generally the treatment is ophthalmic corticosteroids (prednisolone) Psoriatic Arthritis: 1. NSAIDs are the first line of therapy 2. Methotrexate is second line 3. Etanercept is approved for monotherapy Enteropathic Arthritis: 1. NSAIDs 2. 5 – ASA derivatives (Sulfasalazine,pentasa) 3. Biologic medications, the TNF inhibitors
  • 4. 1. Plaque psoriasis (silver scaling, erythematous plaques, guttate lesions, pustules) 2. Psoriatic nail changes 3. Dactylitis (inflammation of the digits, especially the DIPs) Enteropathic Arthritis: Axial or peripheral joint pain in a patient with symptom of IBD (Chronic diarrhea, cramping, mucus in stool, tenesmus, oral aphthous ulcers) Arthritis symptoms may precede the IBD symptoms N/A
  • 5. OSTEOARTHRITIS Chronic, progressive, non- inflammatory , idiopathic, degenerative, joint disease Primarily OA: Closely related to the aging process Other arthropathies can predispose to OA Cause of secondary OA: - WEIGHT!!!!! MOST COMMONLY AFFECTS: - joints of the: - Hands - weight - bearing joints (knees, hips) SYSTEMIC→ - Primary OA is NOT associated with any systemic symptom LOCATION→ - Hands (wrists, MCPs, PIPs, DIPs) - Weight bearing joints (hips, knees) INFLAMMATION→ - *JOINTS ARE NOT EXTERNALLY INFLAMED (NO SIGNS OF INFLAMMATION)!!!* Joint pain is most common presenting complaint. Patient tend to be older (50+) Secondary OA Risk Factor is: - Obesity - Joint trauma - Repetitive joint work - and other arthropathies are risk factors Primary Risk Factor: - Usually Idiopathic Patient’s will often complain of pain in knees, hips, and hands Absence of constitutional symptom (no fever, out of the ordinary fatigue, malaise,etc) *REMEMBER: OA IS NOT AN INFLAMMATORY CONDITION!!* PX: - Joint often appear normal, externally. - Decreased range of motion (ROM) - Crepitus→ osteophytes - Nodes on DIPs and PIPs (both nodes are present) ALL LABS ARE NORMAL *Abnormalities in labs are not indicative of osteoarthritis* The first step in diagnosis is X Ray of the affected joints Typical findings is a reduction in joint space Therapy is palliative: - Reduce pain - Maximize functional capacity. • NSAIDs are the mainstay of treatment !!! (ibuprofen, ketoprofen, meloxicam, diclofenac) • Acetaminophen is the treatment of choice !!! Occupational therapy Weight reduction Hot/cold compresses Exercise Physical therapy
  • 6. CHRONICITY→ - chronic and progressive. - Insidious onset. - Worsens with activity, pain improves with rest EVIDENCE OF TRAUMA→ - May occur secondary to traumas to specific joints. - Primary OA is not associated with trauma
  • 7. RHEUMATOID ARTHRITIS Chronic inflammatory, systemic disease highlighted by severe joint pain. Women > Men Progressive Can dramatically reduce quality of life Location : - particularly in hands, but NOT in DIPS (distal interphalangeal joint) Systemic? YESS!!!!! Most patients will have a history of: - Fatigue - Malaise - Weakness prior to the onset of joint pain - Fever - Fatigue - Malaise Multiple systemic complications: - Anemia of chronic disease (ACD) - Felty’s syndrome (due to splenomegaly, no pancytopenia) - Baker’s cysts Felty's syndrome: - is a complication of long-standing rheumatoid arthritis - defined by the presence of three conditions: - Rheumatoid arthritis - Splenomegaly Technical diagnostic criteria (ACR) Must fulfill at least 4 for diagnosis: 1. Morning stiffness of affected joints for > 1 hr X 6 weeks 2. Swelling of hand joints x 6 weeks 3. Symmetric joint swelling X 6 weeks 4. Subcutaneous nodules (Rheumatoid nodules) 5. Positive rheumatoid factor (RF) 6. Deformities noted on radiography Based on: - Clinical - Imaging - Laboratory criteria There is no one pathognomonic test - positive RF - Anti -cyclic citrullinated peptide (anti-CCP) ( more sensitive but more expensive) *antibodies are highly suggestive* The best initial diagnostic step is radiography (XR is fine, CT is more accurate) *Xray first, CT more expensive* Rheumatoid factor should be drawn as well *Note: if patient has only one inflamed joint or the inflamed joints are asymptomatic, a synovial fluid test may be the best first step* Metotrexate (MTX) *It can cause hepatic fibrosis*, with or without a biologic agent *NSAID, and steroids are not disease modifying* Biologic agents: - Primarily TNF- a inhibitors. - Greatly reduce general inflammation - Infliximab - Etancercept - Adalimumab - Adverse effect: - Immunosupression - Before starting a patient on a biologics get a Tuberculosis test (PPD test) to rule out latent TB Newly diagnosed patients may be treated with an initial short- term course of: - corticosteroids (prednisone), to bridge over, as MTX and biologics take effect NSAIDs can be used for pain relief, but not alter disease course Rheumatoid arthritis is associated with a high risk for morbidity and premature death secondary to the earlier development of cardiovascular, lung diseases and malignancy Boutonniere deformity is a deformed position of the fingers or toes, in which the joint nearest the knuckle (the proximal interphalangeal joint, or PIP) is permanently bent toward the palm while the farthest joint (the distal interphalangeal joint, or DIP) is bent back away (PIP flexion with DIP hyperextension). COMPLICATIONS: - Adverse effects from medications - Progressive reduction of quality of life - Felty’s syndrome - Baker’s cysts - Carpal tunnel syndrome : - Casting - Immobilization - Atlanto-axial subluxation : - patient should be advised to caution anything that involves intense cervical motion - Symptom of impingement vary from hand/ foot paresthesias to quadriplegia
  • 8. - An abnormally low white blood cell count. Baker’s cyst: - is a fluid-filled growth behind the knee. - It causes a bulge and a feeling of tightness. - Also called a popliteal cyst. - A Baker cyst sometimes causes pain. - The pain can get worse with activity or when fully - Straightening or bending the knee - Knee damage caused by a sports injury or a blow to the knee can lead to a Baker's cyst developing. - Rheumatoid arthritis - Osteoarthritis - Gout If a synovial fluid analysis is ran on a patient With Rheumatoid Artheritis there will be: - elevated WBCs not elevated enough to suspect septic arthritis - If suspected, first step is always to stabilize patient and get a plain radiograph of the neck *ACUTE PHASE REACTANTS: AFTER FELTY SYNDROME* Although RA is more common in females, extraarticular manifestations of the disease are more common in males !!! The extra-articular manifestations of RA can occur at any age after onset characterized by destructive polyarthritis and extra-articular organ involvement, including the: - Skin - Eye - Heart - Lung - Renal - Nervous and gastrointestinal systems
  • 9. Baker's cysts usually develop in people aged 30 to 70, although they can affect people of any age, including children.
  • 10. POLYMYALGIA RHEUMATICA ● ALWAYS SEEN IN >55 ● TEMPORAL ARTERITIS CORRELATION ● IDIOPATHIC ● INFLAMMATORY DISORDER OF PAIN ● ASSOCIATED WITH PROXIMAL MUSCLES. *PAIN* ● ABRUPT ONSET ● SELF LIMITED *the key feature is pain, and not too much weakness* Whites are affected more than blacks 2:1. Woman affected more than men There is a high correlation between polymyalgia rheumatica and temporal (giant cell) arteritis. This is a syndrome of the elderly (50+) Nearly half of patients with temporal (giant cell) arteritis have, or will have polymyalgia rheumatica!! Patinent will describe: - General aches and pain - Stiffness - Especially of the upper arms - Shoulders - Hip girdle The pain tends to be worst in the morning PX: - Normal strength (however exerting more strength may elicit pain) - Active range of motion is limited by pain Clinical and of exclusion - The best initial test would be an ESR , which would be elevated * START THE TREATMENT RIGHT AWAY PLEASE* Oral corticosteroids, namely, prednisone Patinet typically have a rapid response to corticosteroid (Then asses if patinet has teporal arteritis via biopsy) The steroids can be tapered down to the lowest effective dose All patients should be assessed for symptoms related to temporal arteritis, due to the high association Patienst with symptomsof temporal arteritis will require: - a temporal artery biopsy for confirmation and consultation with an ophthalmologist. However, the treatment (w/prednisone) is the same Imagining does not play a significant role in diagnosing PR. However, MRI has shown that the inflammation is more around the bursa and tendon, rather, than inside the muscle Itself
  • 11. FIBROMYALGIA Chronic , non- inflammatory , “pain processing” disorder Syndrome of: - Widespread pain - Stiffness - Fatigue - Disrupted sleep Often accompanied by: - Mood or anxiety disorders - Disturbances in memory and other somatic disorders (Irritable bowel syndrome, overactive bladder) Increased incidence in patients suffering from “autoimmune disorders” Increased in patients who have suffered: - Psychiatric or physical trauma (may start disease) Majority of the patients between the ages of 35 and 60 years Symtom/ PX: Patient will appear tired *Pain is illicitable from characteristic “pressure points” which are mandatory for diagnosis !!!* A basic lab profile: - CMP - CBC - TFT - ESR - A mental health exam will be of use ENTIRELY CLINICAL !!! 1. It is prudent to rule out or vigilant for any other alternative/co- existing disorders 2. Symtomatic Treatment All patients diagnosed with fibromyalgia should be prescribed a medication for the pain The most common drug used is a TCA *for pain killers* (amitriptyline) Recently, gabapentin and pregabalin have been used *More effective but more expensive* Sleep medications: - Zolpidem (reduces fibromyalgia fatigue, but not fibromyalgia pain) *Patient requires psychiatric evaluation/ consultation) Patients often complaints of “ hurting all over” “feeling like I always have the flue”, persistent fatigue, co- existence of other somatic problems such as IBS or overactive bladder TRIGGER POINTS: - 18 points (9 pairs) tend to be painful when pressed, and may spread pain to other body parts. People with fibromyalgia have pain in at least 11 of these tender points when a doctor applies a certain amount of pressure
  • 12. Women are more likely to be diagnosed with Fibromyalgia May be difficult to diagnose and is often a diagnosis of exclusion !!!
  • 13. SYSTEMIC LUPUS ERYTHEMATOSUS (SLE) - Chronic, inflammatory, multisystemic disease in which arthritis is one of a myriad of possible symptoms and presentations - Idiopathic - UNKNOWN ETIOLOGY AND SOME GENETIC PREDISPOSITION - Blacks and Hispanics are affected at higher rates than whites - Women are affected more than man by a 9:1 ration - Age of onset: - < 16 years (20%) - >16 to 55 years (65%) - > 55 years (15%) Major causes of mortality: - Infections and nephritis (early) - Atherosclerosis (late) Genetics: - HLA-A1 - HLA-B8 Younger female, often of color presenting with facial rash and/or joint pain Be sure to get OB history Anti - RO antibodies - Fever - Chills - Headache - Fatigue - Malaise - Weight loss Renal: - Hematuria - Protenuria Skin: - Malar “Butterfly” rash - Photosensitivity rash Cardiac: - Myocarditis - Pericarditis - Endocarditis - Atherosclerosis - Pancarditis CNS: - Conginitive dysfunction DIAGNOSTIC CRITERIA FOR SLE (SOAP BRAIN MD) Four diagnostic criteria satisfies diagnosis S- Serositis O- Oral Ulcers A- Arthritis P- Photosensitivity *avoid sunlight =, because it can aggravate and cause B- Blood D/O R- Renal Abnormalities A- ANA+ ⇒ not Gold Standard I- Immunological phenomenon (anti- Sm, AntidsDNA) *GOLD STANDARD* Ultimately a diagnosis based on criteria and supported by laboratory evidence the best initial diagnostic test is ANA *ANA IS NOT THE MOST ACCURATE* Patients who are ANA + should get Anti-dsDNA and Anti- Sm serologies, as they are more specific (HALLMARK) Focused on controlling symotoms Arthralgia/pleuritis/ inflammation: - NSAIDs (Ibuprofen, ketorolac, etc) Rash: - Corticosteroid topical ointment (betamethasone) *Preventative care and corticosteroids (prednisone) should be used when symptoms become frequent and certainly if patient develops renal insufficiency* In severe cases (severe lupus nephritis, heart/lung/ CNS involvement) patients should be started: - Cytotoxins (azathioprine, cyclophosphamide)
  • 14. - HLA-DR *Classic triad: - Fever - Arthralgia - rash (malar, discoid, photosensitive) Pregnancy and SLE: - At high risk for spontaneous abortion - This is a manifestation of anti- phospholipid antibody syndrome - All pregnant woman should be screened during pregnancy for Anti- Ro antibody - Babies born to Ro+ mothers are at risk for neonatal lupus , which typically manifests with a complete heart block - All drug should be stopped during first trimester, After first trimester patient should be started on lowmolecular weight heparin and aspirin NEONATAL LUPUS Pulmonary: - Pleurisy Musculoskeletal: - Myalgias - Arthralgias - Arthritis Hematologic: - Anemia - Leucopenia - Thrompicytopenia - Pancytopenia Lymphatic system: - Lymphadenopathy OB/GYN: - History (Spontaneous Abortions) N- Neurologic symptoms not explained otherwise M- Malar Rash *Butterfly* D- Discoid Rash In emergent flares (RP GMN, severe neuro symptom): - iv corticosteroids and cytotoxics should be used Belimumab is the one biologic that is approved for use in SLE
  • 15. Clinical Manifestation: - Cardiac: - Conduction defects - Complete heart block rhythm abnormalities - Dermatologic: - Erythematous papules - Annular plaques - Hematologic: - Thrombocytopenia - Leukopenia - Anemia - Hepatic: - Hepatitis TREATMENT: - Involves: - Surgical implantation of a pacing device - Symptomatic control - Generally with hydrochloroquine and IV
  • 16. or topical steroids NSAIDS SHOULD BE AVOIDED => MAY DEVELOP RAYE’s SYNDROME RESOLVES BY 6-8 MONTHS OF AGE!!
  • 17. MYOSITIS ( POLYMYOSITIS & DERMATOMYOSITIS) Idiopathic (slow- onset, progressive), autoimmune, inflammatory disease of skeletal (and occasionally cardiac) muscle Idiopathic myositis (Polymyositis, Dermatomyositis, Inclusion body myositis): - Age of onset is around 45-60 years - Increased incidence with other autoimmune diseases - Best first diagnostic step is: - Creatine phosphokinase (CPK) and Aldolase - Most accurate test is: - biopsy and it is absolutely necessary to establish the diagnosis POLYMIOSITIS & DERMATOSIOTIS: Women are affected more than men 2:1, Blacks more than whites 5-8:1
  • 18. *For inclusion body myositis men affected more than woman* POLYMYOSITIS: 1. Idiopathic, autoimmune and inflammatory 2. Affects proximal muscles most dramatically 3. Symmetrical 4. Distal muscles are relatively conserved 5. Ocular muscles are NEVER involved 6. Prognosis is good: main complications are: a. dysphagia b. interstitial lung disease *A high creatinine kinase result would suggest Polymyositis, CK is released when muscles breakdown which occurs in polymyositis* *Symmetrical muscle weakness affecting big muscles is seen in polymyositis* Polymyositis: - Progressive Weakness - Problems with ADL (activities of daily living) including: - brushing hair - getting out of a chair - holding head up - carrying objects - lifting arms up over head Symptoms will be worst in the morning, and improve throughout the day May present: Weight loss Fever Hyperhydrosis Generalized hyperhidrosis Chronic pain POLYMYOSITIS: 1. best first step in the management/diagnosis: a. CPK and aldolase levels *These are markers for muscular inflammation* 2. The most accurate test is: a. muscle biopsy *Do this if CPK/Aldolase are high* 3. It is judicious to order titers for Anti- Jo1 a. Patient + for this antibody are at increased risk to develop interstitial lung disease POLYMYOSITIS: Best initial treatment for polymyositis is: - Corticosteroids, (prednisone), If prednisone alone is not effective, cytotoxics or biologics may be added ADULT POLYMYOSITIS: Rituximab DERMATOMYOSITIS: Rituximab in the Treatment of Refractory Adult and Juvenile Dermatomyositis
  • 19. DERMATOMYOSITIS: Idiopathic, autoimmune and inflammatory disease Also affects the skin. 1. Affects proximal muscles most dramatically 2. Symmetrical 3. Distal muscles are relatively conserved. 4. Ocular muscles are NEVER involved INCLUSION BODY MYOSITIS: Hereditary or acquired inflammatory and pathologic disorder of muscle. Inclusion body myositis may be asymmetrical Affect distal muscles (hands, fingers) more notably Fatigue Dyspnea Malaise DERMATOMYOSITIS: *same as polymyositsi* - Progressive Weakness - Problems with ADL (activities of daily living) including: - brushing hair - getting out of a chair - holding head up - carrying objects - lifting arms up over head May present: Weight loss Fever Hyperhydrosis Generalized hyperhidrosis Chronic pain Fatigue Dyspnea Malaise 1. The best first step is: a. CPK and aldolase levels. 2. The most accurate test is: a. muscle biopsy (do this if CPK/Aldolase are high) Inclusion Body Myositis: - Is refractory to steroids and immunosuppressants. - Not deadly - - Early stages of disease can be confused w/ amyotrophic lateral sclerosis. The muscle biopsy will confirm Inclusion body myositis Unlike Inclusion body myositis amyotrophic lateral sclerosis is fatal
  • 20. Affects men more than woman In addition, dermatomyositis will present with the characteristic heliotropic rush (face, eyelids, sunexposed areas) 2 SIGNS & 1 RASH : GOTTRON’s SIGN: - purple/pink papule over the Metacarpophalangeal (MCP) and proximal interphalangeal (PIP) joints. HELIOTROPE RASH: - purple/pink rash around the eyes - eyelids may be swollen. SHAWL SIGN: - rash over the upper back, shoulders and a V on the chest INCLUSION BODY MYOSITIS: INCLUSION BOSY MYOSITIS 1. The best initial diagnostic step:
  • 21. - Insidious asymmetrical weakness: - Dysphagia - Distal and proximal weakness, - Droopy head - Facial muscle weakness - No ocular involvement Px: Decreased tendon reflexes are common a. CPK and aldolase 2. The most accurate test is: a. Biopsy *Electromyography (EMG) may be ordered*
  • 22. WEGENER’S GRANULOMATOSIS Auto-immune inflammatory systemic disease of the SMALL VESSELS!! Commonly implicated by C- ANCA Involves: Organ System, but most prolly: Respiratory tract Skin Kidneys History of chronic sinusitis that is refractory to therapy Refractory chronic sinusitis “Saddle nose” deformity Hemoptysis Dyspnea Polyarticular arthritis Palpable purpuric rash N/A Elevated ESR Elevated CRP Normocytic Anemia Abnormal UA The best initial test: an ANCA (c-ANCA) titer The most accurate test is: Renal Biopsy *shows granulomas* Prednisone and cyclophosphamide are the mainstays of therapy After 18 months, Cyclophosphamide ( which cannot be used more than 18 months) can be replaced with Methotrexate or Azathioprine Among the vasculitis in that the upper respiratory symptoms tend to take central stage Note: the lungs will always be involved even if the patient is asymptomatic. CXR or CT (more accurate but may not be necessary) will show nodules *ONLY CYCLOPHOSPHAMIDE REPLACES* BONUS: What is an adverse effect of Cyclophosphamide? Patient’s who are on cyclophosphamide therapy should get prophylaxis for Pneumocystis carinii pneumonia • TMP/SMX or (dapsone or atovaquone)
  • 23. MICROSCOPIC POLYANGIITIS Auto- immune inflammatory systemic disease of the SMALL VESSELS!! Commonly implicated by P-ANCA Can involve any organ system, but most prolly involved are: Skin Lungs Kidneys Neuromuscular system *MULTISYSTEMIC* Most patient’s will describe a history of frequent: Muscle aches and pains Fatigue Weakness Malaise Weight loss Hemoptysis Mononeuritic multiplex: (Inflammation of two or more nerves, typically in unrelated parts of the body) Palpable purpuric rash: N/A Elevated ESR Elevated CRP Normocytic Anemia Abnormal UA The best initial test: an ANCA (p- ANCA) titer The most accurate test is biopsy NOT IN PPT????? Note: Microscopic polyangiitis can be conclusively differentiated from Wegener’s granulomatosis on biopsy No granulomas SEEN POLYARTERITIS NODOSA Autoimmune inflammatory systemic disease of the MEDIUM VESSELS, particularly those in the muscle Can involve any organ system, but uniquely PAN spares the lungs (NEVER INVOLVED) Most commonly involved: Skin Kidney Nerves N/A Neuro/neuromuscular symptoms are most prominently present in 70% of patient which includes: Peripheral neuropathy (pins and needles) Transient loss of vision Patient must have 3 of the following 10 criteria : 1. Weight Loss of 4 KG or more 2. Livedo reticularis (Mottled skin) 3. Testicular pain or tenderness 4. Myalgia or leg weakness/tenderness Elevated ESR Elevated CRP Normocytic Anemia Abnormal UA Best initial diagnosis & most accurate test is a biopsy *This can be done at a skin ulcer, at the sural nerve or skeletal muscle.* *If a biopsy cannot be undertaken, angiogram is ok (i.e renal angiogram). Prednisone and Cyclophosphamide *Postprandial GI pain is not uncommon* *Hypertension secondary to renal factors is common too (restricted arterial flow to kidneys activates RAAC* *In patients with HBV & HCV, cyclophosphamide CAN NOT BE USED!!!* Cyclophosphamide may be substituted with Methotrexate or Azathioprine in 18 months (remission
  • 24. GI tract (Postprandial abdominal pain) Associated with: Hep B Hep C Mononeuritis multiplex (Inflammation of two or more nerves, typically in unrelated parts of the body) The skin rash is similar to WG and MPA, however it may also include livedo reticularis and Raynaud phenomenon (due to the fact that this is a medium vessel disease) Significant weight loss Men: testicular pain or tenderness 5. Mononeuropathy or polyneuropathy 6. Diastolic BP> 90 mmHg 7. Presence of HBsAg in serum 8. Arteriogram demonstrating aneurysms or occlusions of visceral arteries (transient loss of vision) 9. Biopsy of small or medium- sized artery showing polymorphonuclear leukocytes (PMNs) in wall FOR CONCLUSIVE DIAGNOSIS, THE PATIENT MUST HAVE BIOPSY!! Angiogram will show multiple aneurysms* CHANG-STRAUS SYNDROME Autoimmune, inflammatory systemic disease that is uniquely associated with asthma and eosinophilia Like Wegener’s granulomatosis: Asthma Recurrent sinusitis Constitutional sx Peripheral neuropathy Mononeuritis multiplex GI pain ACR criteria: Patient must have 4 of 6 criteria - Diagnosis (asthma) - Eosinophilia (>10 % in peripheral blood) Elevated Eosinophil count (>10 %) Elevated ESR, Elevated CRP In the presence of clinical suspicion Initial diagnostic step is a biopsy of an affected area (skin, renal, muscle, lung, nerve) Prednisone alone is sufficient, unless the symptoms are life threatening *Higher rate of coronary involvement (aneurysms)* WAY TO REMEMBER THIS DISEASE LOL: *If Wegener’s disease and Polyarteritis Nodosa got together and had a bastard child, it’s name will be Chung- Strauss, and he’d have asthma*
  • 25. - Has chronic sinus and nose involvement (usually doesn’t perforate though) - Involves the small vessels - Can cause glomerulonephritis (UA=RBCs++) - Tends to be ANCA positive (but it’s p-ANCA) - Granulomatous on biopsy Like Polyarteritis Nodosa : - Has neuromuscular involvement - Can develop mononeuritis multiplex, peripheral neuropathies - Postprandial abdominal pain is a common complaint Unique to Chang- Straus : - Patient ALWAYS has asthma or some kind of reactive airway syndrome - Patient has eosinophilia (>10 %) Cough Arthralgias Hemoptysis (remember, PAN spares the lungs >_< ) Hematuria Purpuric rash - Paranasal sinusitis - Mononeuritis multiplex or polyneuropathy - Pulmonary infiltrates (on CXR or CT) - Histological proof of vasculitis w/extravascular eosinophils ACD present UA will generally be abnormal showing a GMN picture (RBCs++, RBC casts) p- ANCA+ Chung- Straus shares features of both Wegener’s disease and Polyarteritis Nodosa and the patient with Chung- Strauss always has asthma
  • 26. - Biopsy shows eosinophils in granuloma TEMPORAL ARTERITIS (GIANT CELL ARTERITIS) Inflammatory disease of the vessels involving the medium and large arteries of the head/scalp (branches of the ECA, most prominently the superficial temporal artery) Occurs always only in patient over the age of 55 Painful temples Pain on chewing Tender scalp Visual disturbances: - Diplopia - Blurry vision May co-exist with polymyalgia rheumatica (proximal muscle pain) PX: - Scalp tenderness - Superficial temporal artery is visibly inflamed/engorged on the temple or lateral forehead N/A The best initial diagnostic step is to get an erythrocyte sedimentation rate (ESR) Most accurate test: temporal artery biopsy *If the ESR is elevated, proceed straight to treatment (Prednisone)* You must also get a temporal artery biopsy for confirmation, as it is the most common test. Prednisone
  • 27. TAKAYASU’s ARTERITIS (PULSELESS DISEASE) Inflammatory disease of the LARGE VESSELS!! ( implicating the aorta and usually its initial ascending branches (L Brachiocephalica, R Common Carotid , R Subclavian a), however, it may descend all the way to the aortic bifurcation) Most commonly affects women of childbearing age ( <40 years). More prevalent in Asians Variable and non- specific Headache Malaise Arthralgia Visual deficits. Claudication of the extremities Stoke TIA and seizures are rare manifestations PX: - Very important for establishing suspicion - Carotid or subclavian bruit - Aortic regurgitation - Weak upper extremity pulse - BP difference between arms of Diagnosis is likely if pt has 3 of the 6 criteria : 1. Younger than age 40 2. Extremity claudication 3. Decreased pulsation of one or both brachial arteries 4. Difference in systolic BP of >10 mmHg between arms 5. Subclavian bruit or abdominal aorta bruit 6. Arteriographic evidence The most accurate test is arteriography Prednisone is the first line of therapy Cyclophosphamide or Methotrexate may be added if there is no significant response to prednisone alone *Very important to ensure normal BP and manage other cardiovascular risk factors, especially in pregnancy*
  • 28. >10 mmHg BEÇHET’s SYNDROME Inflammatory, multisystemic disease of SMALL VESSELS!! resulting in frequent aneurysm and rupture Idiopathic Affects: - Eyes - Mucous membranes - Genitals *recurrent aphthous ulcers* Recurrent aphthous ulcers Painful genital ulcers Eye pain w/redness (uveitis) Rash (erythema nodosum acne like rash) Rarer but more serious symptom include: Meningoencephalitis GI bleed Amyloidosis ISG Criteria for Behcet’s Syndrome: *MUST HAVE* - Recurrent oral aphthous ulceration (3x in 12 month period) * AND 2 OF THE FOLLOWING* - Recurrent genital ulceration or scarring - Eye lesions: - Anterior uveitis - Posterior uveitis, - Retinal vasculitis (dx by ophto) - Skin lesions: - Erythema nodosum - Acneiform nodules - Papulo-pustular lesions (in patient who has no other explanation) BASED ON SYMPTOMS!!!!! ORAL ULCERS: (Topical corticosteroids, colchicine) GENITAL ULCERS: (Topical corticosteroids) UVEITIS: (Topical corticosteroids) RETINAL VASCULITIS: Cyclosporine
  • 29. - Positive patherge test: - Insert hypodermic needle into skin, after 48 hours, papule present in the region KAWASAKI DISEASE Is also known as mucocutaneous lymph node Syndrome Is an acute self limited febrile illness of infants and children (< 5 yrs) JAPAN, HAWAII ( ENDEMIC) KD is a childhood vasculitis that mainly targets coronary arteries Coronary artery involvement: – can lead to coronary thrombosis or aneurysm formation and its rupture Features: Oral Erythema Palmar Erythema Conjunctivitis Rash Desquamation Edema: FEET & ARMS Clinical Findings: - High fever – Erythematous rash of trunk and extremities with desquamation of skin. N/A Neutrophilic leukocytosis Thrombocytosis : * characteristic finding* Elevated ESR Abnormal ECG *acute MI*
  • 30. – Mucosal inflammation : cracked lips, oral erythema – Erythema, swelling of hands and feet. – Localized lymphadenopathy (cervical adenopathy) – MCC of an acute MI in children**** BUERGER’s DISEASE Also known as Thromboangitis Obliterans. Is a peripheral vascular disease of smokers. Pathology: Earliest change: - Acute inflammation involving the small to medium sized arteries in the extremities (tibial, popliteal & radial arteries). Inflammation of vessel: - Thrombus formation Clinical Findings: - Young-middle age, male, heavy smoker ( Is***l, Japan, India) Symptoms start between 25 to 40 years: - Early manifestation: - Intermittent Claudication in feet or hands. (Cramping pain in muscles after exercise, relieved by rest) - Late manifestation: N/A BIOPSY Early stages of vasculitis frequently cease on discontinuation of smoking.
  • 31. - Obliterates lumen - Ischemia - Gangrene of extremity Inflammation also extends to adjacent veins and nerves. (Involvement of entire neurovascular compartment) - Painful ulcerations of digits - Gangrene of the digits often requiring amputation. HENOCH SCHONLEIN PURPURA (HSP) A variant of hypersensitivity vasculitis. Seen in children (MC vasculitis in children) Rare in adults. Usually occurs following an upper respiratory infection Caused by deposition of IgA-C3 immune complexes in vessel wall. Vessels involved: - Arterioles Clinically characterized by: – - Palpable purpura over extensor aspects of arms and legs. - commonly limited to lower extremities/ buttocks. Involvement of: ● GIT ○ colicky abdominal pain, melena ● Musculoskeletal system: N/A Neutrophilic leukocytosis Deposition of IgA-C3 immune complexes in: Skin Renal lesions STEROIDS
  • 32. - Capillaries and venules of: - Skin - GIT - Kidney - Musculoskeletal system. ○ Arthralgia (non migratory) ○ Myalgias ● Kidneys: ○ hematuria due to focal proliferative GN ● Lung: ○ Rare INFECTIOUS VASCULITIS Fungal vasculitis: vessel invading fungi - Mucor - Aspergillus - Candida Rocky Mountain spotted fever: - Rickettsia Disseminated meningococcemia: - Small vessel vasculitis N/A
  • 33. - petechial hemorrhages Infective endocarditis: - Roth’s spots in retina - Janeway’s lesions on hands (painless) - Osler’s nodes on hands (painful) - Glomerulonephritis SJÖGREN’s SYNDROME Autoimmune, inflammatory and destructive disease of the exocrine organs (primarily parotid glands, lacrimal ducts, etc) Affects 0.1- 4 % of the US population Female is preponderance Primary and secondary forms History of numerous dental caries, due to chronically dry mouth In many cases, the patients will have a history of another autoimmune disease (SLE, RA, etc) as many as 1/3 of RA patients also have Sjogren’s syndrome The major symptoms are: - Dry eyes (xerophthalmia) - Dry mouth (xerostomia) - Parotid gland enlargement Typical presentation : - dry eyes / dry mouth in female patients Pts must have 2 of the following 3: Ocular staining score at least 3 Biopsy of salivary gland (single most accurate test) showing lymphocytic sialadenitis Positive ANA . Specifically Anti-Ro and Anti-La CRITERIA BASED *Biopsy of salivary gland (single most accurate test* Schirmer’s test: - This test consists of placing a small Lymphoma is a serious complication of SS, occurring LATE in the disease. Risk of lymphoma is 44 times greater than the general population 20-30% of patients have vasculitis. Renal involvement in 10% of patients interstitial nephritis
  • 34. Complications: - NonHodgkin’s lymphoma - Neonatal lupus - Some degree of parotid gland swelling may be present Mild to moderate sensorineural hearing loss in high frequencies. *Anti-cardiolipin antibodies has been demonstrated, but no cochlear or vestibular pathology has been found* Nodes: - Granulomatous and non-granulomatous laryngeal nodes - Bamboo nodes - whitish or yellowish transverse submucosal lesions localized to middle third of vocal fold. Rare first symptom: - Hoarseness Lacrimal-gland biopsy - Focal lymphocytic sialadenitis in minor salivary glands containing more than 50 lymphocytes per 4 mm of glandular tissue strip of filter paper inside the lower eyelid (conjunctiva sac). - The eyes are closed for 5 minutes. - The paper is then removed and the amount of moisture is measured. - A young person normally moistens 15 mm of each paper strip. - Because hyperlacrimation occurs with aging, 33% of normal elderly persons may wet only 10 mm in 5 minutes - Persons with Sjögren's syndrome moisten less than 5 mm in 5 minutes RESULTS OF SCHIRMER TEST!!!! 1. Normal: - which is ≥15 mm wetting of the paper after 5 minutes. Renal tubular acidosis also seen
  • 35. Thyroid abnormalities: - “Hashimotos’s thyroiditis” can be found in up to 1/2 of Sjögren’s syndrome patients - 10-15% of Primary Sjögren’s syndrome are clinically hypothyroid - Therefore, all patients with this syndrome should undergo routine evaluation of thyroid function 50% having nasal mucosa hypertrophy 13% have septal perforations. Pulmonary Symptoms: - Xerotrachea (dry cough) - Dyspnea 2. Mild: - which is 14-9 mm wetting of the paper after 5 minutes 3. Moderate: - which is 8-4 mm wetting of the paper after 5 minutes. 4. Severe: - which is < 4 mm wetting of the paper after 5 minutes.
  • 36. Esophageal spasm and dysmotility are also seen, but may be secondary to absence of saliva, - Atrophic gastritis - celiac disease GOUT -- Crystal deposition in the joint--- - inflammation, pain -- Acute pain attacks -- Men affected more than women -- Blacks more than whites. -- Average age of initial onset is 30-60 years *Most often in early patients* -- Urate crystal deposition in the joint, resulting in tends to occur in older men( RF= age): increased risk in: - Overweight - Alcohol drinkers - Patients on diuretics (loop ( furosemide) and thiazide) Excruciating monoarticular joint pain *may wake up the patient* Overlying erythema visible swelling occasionally there may be nodules in the soft tissue. *Inflamed joints* Fever may be present if there is “polyarticular” involvement N/A The best initial diagnostic step is: synovial fluid analysis (arthrocentesis) *don’t need to do every time* Maybe diagnosing clinically if the patient has had several confirmed attacks of gout and this episode is similar The best initial step in therapy is NSAIDs *We treat the pain first !* To prevent further gout attacks, patients should be started on: - Allopurinol: - Usual dose is 300 mg/day. The Maximum recommended dose is 800 mg/day. - Dose depends on Kidney function: - GFR - Check creatinine - In renal insufficiency dose should be decreased to 200 mg/day for creatinine clearance < 60ml/min and to 100 mg/day if clearance < 30 ml/min). - Start with small doses of allopurinol to reduce the risk of precipitating an acute gout attack. - Most common side effects are: Maybe mistaken for cellulitis (over the affected joints) Hallmark: Only in Gout Patients
  • 37. inflammation and pain. -- Most commonly first occurs in the big toe (significant pain will occur too) ● May also occur in the: ○ Ankle ○ Knee ○ Proximal Interphalangeals (PIPs) ○ Distal Interphalangeals (DIPs) ○ Maybe polyarticular (fever also) *monoarticular = no fever* - Rash (2% of patients) - rarely patients can develop exfoliative dermatitis that can be lethal Colchicine may be used as well, but it has “significant GI effects” Febuxostat is another, more recent drug on the market Patients with multiple gout attacks should undergo: - A 24- hour uric acid collection *Do not fall for the trap of ordering serum uric acid levels !!! They don’t correspond to gout attacks!*
  • 38. PSEUDOGOUT Deposition of calcium pyrophosphate dehydrate, calcium oxalate or calcium hydroxyapatite crystals in the joint, resulting inflammation and pain Tends to occur in older patients with pre- existing joint disease and in patients with metabolic or electrolyte disorders (particularly involving calcium) *Hypercalcemia, Hypophosphatemia, Hyperparathyroidism, Hemochromatosis, Hypomagnesemia* Presentation is almost identical to gout not GOAT ( lol >_<) Most common joint affected is the knee. The wrist, ankle, and shoulder may also get affected N/A Uric Acid: normal values ranges: - from 4.0 to 8.6 mg/dl in men - from 3.0 to 5.9 mg/dl in women. Urinary levels are normal below 750 mg/24h Urinary levels above 750 mg/dl in 24h in gout or > 1100 mg/dl in asymptomatic hyperuricemia indicates urate overproduction The best diagnostic test is: Synovial fluid analysis (arthrocentesis) Like gout, the best initial therapy is NSAIDs *Consider workup for underlying cause* If Comprehensive Metabolic Panel (CMP) shows elevated calcium levels, consider getting a PTH level May put patient on low dose colchicine as a prophylactic for future attacks. Diet is usually impractical, ineffective and rarely adhered to in clinical practice ( for both ) SCLERODERMA (CREST SYNDROME & SYSTEMIC SCLEROSIS) Scleroderma is a symptom (thickening of the skin) A term that can refer to two similar but separate clinical syndromes Almost all (more than 90%) of patients with scleroderma also have Raynaud's phenomenon. Systemic Sclerosis: ACR criteria for Systemic sclerosis Patients must have one major criteria or two minor criteria CREST SYNDROME: Clinical, major differential is systemic sclerosis Systemic Sclerosis: - Symptomatic. Major focus is in preventing scleroderma renal crisis (malignant hypertension) and minimizing pulmonary hypertension Pulmonary manifestations of systemic sclerosis occur in more than 70% of
  • 39. Systemic sclerosis *aka diffuse systemic sclerosis* → CREST + MULTI ORGAN Systemic sclerosis and CREST syndrome both involve cutaneous symptoms (skin thickening, calcinosis, Raynaud syndrome, histological changes) “Systemic Sclerosis” can affect the: - Kidneys (renal insufficiency) - The heart (CHF) - Lungs (pulmonary fibrosis) *CREST SYNDROME DOES NOT AFFECT* Systemic sclerosis is: - CREST syndrome PLUS Multiple internal organ involvement Idiopathic and Female predominance Can have many different presentations CREST syndrome *aka limited systemic sclerosis, cutaneous sclerosis* - Generally have symptoms similar to CREST syndrome - is not limited to peripheral extremities and often affects the face and neck. - Other non- specific symptoms may be present: - Palpitations - Shortness of breath - Fatigue - Weakness Major criteria: - Proximal scleroderma Minor criteria: - Sclerodactyly - Digital pitting scars or loss of substance from finger pad - Basilar pulmonary fibrosis Patients with CREST syndrome should have titers ran for ANA (90% of patients will be+) and anti- Scl- 70 Systemic Sclerosis: Clinical, supported with laboratory evidence Patients + for Anti- Scl-70 are more likely to have diffuse systemic sclerosis Treating CREST and Systemic sclerosis: - Focused on symptoms and in systemic sclerosis, preventing scleroderma renal crisis, and limiting any other complications - Skin thickening : D-penicillamine - Raynaud attacks: Calcium channel blockers (nifedipine) *sometimes a-blockers & Nitrates* - Scleroderma renal crisis (ACE inhibitors) - GI symptoms: Proton- pump inhibitors manage closely for Barrett’s esophagus PULMONARY: - Most common SSc-related cause of death. - Pulmonary diffuse fibrosis is a common cause of severe restrictive lung disease - CXR shows interstitial thickening, most easily seen in the lower lung fields. - ECG→ Right ventricular hypertrophy *Some patients with interstitial lung disease develop slowly progressive patients In Diffuse systemic sclerosis: - Interstitial lung disease (ILD) more common In Limited systemic sclerosis: - Pulmonary hypertension is more common The pulmonic component of the second heart sound is accentuated, and ultimately, right-sided cardiac failure develops. Diagnosis is confirmed by: - echocardiogram or by right heart catheterization.